You are on page 1of 24

Heart Failure

By Adam Sieg, Pharm.D., BCPS

Reviewed by Orly Vardeny, Pharm.D., FCCP, FAHA, FHFSA, BCACP; Stephen B. Vickery, Pharm.D., BCPS;
and Kevin Ordons, Pharm.D., BCPS, BCCP

LEARNING OBJECTIVES

1. Apply treatment strategies to reduce the progression of heart failure (HF) through assessing functional class.
2. Evaluate and justify traditional and newer treatment strategies for patients with stage C HF, specifically as they relate to
the 2017 guideline updates.
3. Develop treatment strategies for patients with stage D HF.
4. Evaluate treatment strategies for potential benefit/harm for the patient with HF with preserved ejection fraction.

INTRODUCTION
ABBREVIATIONS IN THIS CHAPTER
Current Status of Heart Failure
ACC/AHA
American College of Cardiology/
American Heart Association Statistics
AF Atrial fibrillation The prevalence of heart failure (HF) continues to be a significant bur-
ARNI Angiotensin receptor-neprilysin den, not only for the families and individuals who take care of these
inhibitor
patients but also for the health care system as a whole. With a reported
BNP B-type natriuretic peptide
915,000 new cases of HF diagnosed annually, 6.5 million Americans
CRT Chronic resynchronization therapy
20 years and older had a diagnosis of HF in 2011–2014 (Benjamin
CV Cardiovascular
2018). Despite significant efforts to identify and treat the underlying
GDMT Guideline-directed management
risk factors that may prevent HF, projections suggest that by 2030,
and therapy
HF cases will increase by 46%, totaling 8 million people (Heidenreich
HF Heart failure
2013). About 1%–2% of the adult population (in developed countries)
HFpEF Heart failure with preserved ejec-
tion fraction has a diagnosis of HF, which increases with advanced age (Ponikow-
HFrEF Heart failure with reduced ejection ski 2016). Patients of African American heritage have the highest risk
fraction of developing HF, followed by those of Hispanic/Latino ethnicity and
ICD Internal cardiac defibrillator whites. African American patients are believed to be at high risk of
LV Left ventricular developing HF because of a lower socioeconomic status and higher
LVAD Left ventricular assist device prevalence of risk factors (Ponikowski 2016).
MRA Mineralocorticoid receptor The economic burden of HF is of vast concern, with a reported
antagonist cost of $30.2 billion in 2012 and an anticipated cost increase by
RAAS Renin-angiotensin-aldosterone almost 130%, reaching $69.7 billion by 2030 (Heidenreich 2013).
system Efforts are under way to further explain risk factors and implement
strategies to manage risk factors and associated disease preven-
Table of other common abbreviations.
tion. However, despite an improved understanding of HF pathophys-
iology, risk factors, and management strategies, data are conflicting
on overall survival after an HF diagnosis. Compared with other termi-
nal diseases, HF survival remains unacceptably low. According to an
Oxford report, cancer survival has doubled, whereas HF survival has
not changed over the same period (Taylor 2017).
Ideally, a risk stratification score could be applied to an individ-
ual patient to help guide clinicians in treatment as well as to provide

PSAP 2019 BOOK 1 • Cardiology 89 Heart Failure


clearer expectations for family members. However, the many (ACC/AHA) developed a staging system to help pharmacists
prognostic scores developed in the past decades have shown and other health care professionals focus on preventive strat-
only moderate accuracy in predicting mortality (Rahimi 2014). egies in patients at high risk and implement pharmacother-
apy plans once structural damage has developed (Table 1).
Distinguishing Between the Differences in HF The New York Heart Association (NYHA) functional clas-
ACC/AHA and NYHA Recommendations sification provides a complimentary scale to the ACC/AHA
Heart failure is commonly associated with a clinical syndrome system by describing HF severity on the basis of activity lim-
that is highlighted by signs (elevated jugular venous pressure, itation (Yancy 2017). Unlike the ACC/AHA system, the NYHA
crackles, edema) or symptoms (swelling, shortness of breath, classification has shown a strong relationship between
fatigue). Development of cardiac structural and/or functional functional class, mortality, and quality of life in this patient
abnormalities typically leads to reduced cardiac output or ele- population.
vated intracardiac pressures at rest and during activity, with
HF Updates According to the 2017 Guidelines
subsequent HF symptoms (Ponikowski 2016). The American
College of Cardiology and the American Heart Association Biomarkers for Prevention, Diagnosis,
and Prognosis
Beyond HF pharmacotherapy, new recommendations focus
BASELINE KNOWLEDGE STATEMENTS on preventing, using early evaluation, and delaying the devel-
opment of HF through assessing biomarkers and managing
Readers of this chapter are presumed to be familiar comorbidities (Yancy 2017; Ponikowski 2016). The B-type
with the following: natriuretic peptide (BNP) and atrial natriuretic peptide are
• General knowledge of the pathophysiology that produced in response to high ventricular filling pressures to
leads to heart failure bring about diuretic, natriuretic, and hypotensive responses
• Understanding of the NYHA functional classifica- in addition to inhibition of the systemic sympathetic ner-
tion and the ACC/AHA staging system
vous system and the renin-angiotensin-aldosterone system
• Drug knowledge of traditional HF therapies, includ- (RAAS). Ideally, clinicians could use biomarkers in “at-risk”
ing ACEIs, ARBs, MRAs, and β-blockers
patients (stage A) to help prevent structural heart damage
• Treatment options for patients with HFrEF and HFpEF from becoming overt stage C symptoms. The STOP-HF is the
largest study to date to evaluate BNP as a potential screening
Table of common laboratory reference values
tool for patients with stage A HF (Ledwidge 2013). The com-
posite end point of asymptomatic left ventricular (LV) dys-
ADDITIONAL READINGS
function with or without newly diagnosed HF was reduced in
The following free resources have additional back- patients screened with BNP. Given the limited supporting evi-
ground information on this topic: dence for BNP monitoring as a preventive measure, the guide-
• Yancy CW, Jessup M, Bozkurt B, et al. 2017 ACC/ lines recommend incorporating this strategy into a treatment
AHA/HFSA focused update of the 2013 ACCF/AHA plan that includes consultation with a cardiovascular (CV)
guideline for the management of heart failure: a specialist through optimizing guideline-directed manage-
report of the American College of Cardiology/Amer- ment and therapy (GDMT) (Yancy 2017; Ponikowski 2016).
ican Heart Association Task Force on Clinical Prac- In healthy patients, BNP and N-terminal-proBNP
tice Guidelines and the Heart Failure Society of
(NT-proBNP) concentrations are often similar but should not
America. Circulation 2017;136:e137-61.
be used interchangeably. When LV dysfunction develops, the
• Ponikowski P, Voors AA, Anker SD, et al. 2016 ESC relative concentrations of both increase, with higher values
guidelines for the diagnosis and treatment of acute
and chronic heart failure: the task force for the in NT-proBNP. Commercially available assays vary, and insti-
diagnosis and treatment of acute and chronic heart tution-derived thresholds should be used when implement-
failure of the European Society of Cardiology (ESC). ing them into practice. In the Breathing Not Properly study,
Developed with the special contribution of the patients with clinically diagnosed HF had significantly higher
Heart Failure Association (HFA) of the ESC. Eur J BNP concentrations than did those without HF (675 vs. 110 pg/
Heart Fail 2016;18:891-975. mL, respectively) (Maisel 2002). A BNP greater than 100 pg/
• Thompson PL, Davis TME. Cardiovascular effects mL correlated with an HF diagnosis with 90% sensitivity, 76%
of glucose-lowering therapies for type 2 diabetes.
specificity, and 83% accuracy. Of note, elevated BNP concen-
Clin Ther 2017;39:1012-25.
trations should be used in the context of the complete clinical
• Anderson SL, Marrs JC. A review of the role of the scenario, not in isolation, especially given the other cardiac
pharmacist in heart failure transition of care. Adv
Ther 2018;35:311-23. and non-cardiac causes of BNP elevation. The current guide-
lines state that BNP and NT-proBNP biomarkers can help sup-
port a diagnosis but may be more useful to rule out HF.

PSAP 2019 BOOK 1 • Cardiology 90 Heart Failure


Table 1. ACC/AHA Staging System and NYHA Functional Classification

ACC/AHA HF Staging System NYHA Functional Classification


Stage Description Functional Class Description

A Patients at high risk of developing HF but No associated N/A


without structural heart disease functional class
• HTN, DM, CAD, metabolic syndrome

B Patients with structural heart disease but no I No limitation of physical activity


signs or symptoms of HF
• Previous MI, low EF, no symptoms

C Patients with structural heart disease and


current or previous symptoms
II Slight limitation of physical activity
• Low EF, HF signs/symptoms

III Marked limitation of physical activity

IV Unable to carry on any physical activity


without discomfort
D Patients with symptoms despite maximal
medical therapy
• End-stage HF

CAD = coronary artery disease; DM = diabetes mellitus; EF = ejection fraction; HF = heart failure; HTN = hypertension; N/A = not
applicable.
Information from: Yancy CW, Jessup M, Bozkurt B, et al. 2017 ACC/AHA/HFSA focused update of the 2013 ACCF/AHA guideline for
the management of heart failure: a report of the American College of Cardiology/American Heart Association Task Force on Clinical
Practice Guidelines and the Heart Failure Society of America. Circulation 2017;136:e137-61.

Beyond its potential diagnostic capabilities, BNP can serve medications for diabetes (e.g., sulfonylureas, thiazolidinedi-
as a prognostic marker in both patients with chronic HF and ones) or arthritis (e.g., NSAIDs) can exacerbate HF symptoms.
patients with acute decompensated HF. A systematic review Pharmacists treating patients with HF must actively review
using BNP as a prognostic marker showed that for every patients’ medications to avoid unwarranted complications.
100-pg/mL increase in BNP, the relative risk of mortality was Anemia is common in patients with HF, but its overall
increased by 35% (Doust 2005). However, even GDMT may not prevalence varies within the literature. Symptoms associ-
offset the risk of mortality in patients with persistently ele- ated with anemia commonly parallel those associated with
vated BNP concentrations. HF (dyspnea and fatigue) and occur because of reduced oxy-
A new recommendation from the ACC/AHA 2017 guideline gen delivery to tissues. Although the cause of anemia is likely
pertains to the use of pre-discharge BNP concentrations as multifactorial, it may include decreased intake and/or absorp-
a prognostic marker for rehospitalizations or risk of death. tion of iron, creating iron-deficiency anemia. Alterations in GI
Studies have shown a strong correlation between patients absorption can result from intestinal edema, use of medica-
with higher BNP concentrations at discharge and those who tions altering gastric pH (e.g., proton pump inhibitors or his-
did not have a reduction in BNP concentrations with respect tamine-2 antagonists), or ingestion of foods that may reduce
to subsequent mortality or readmission (Logeart 2004). absorption (e.g., calcium, tannins, phosphates, antacids).
Despite the potential usefulness of BNP concentrations as a Controversy continues to surround the role of inflammation
prognostic marker, the current guidelines posit that targeting in chronic diseases because of conflicting data. Of particular
certain concentrations or a relative change may be unpracti- importance are the negative clinical consequences that stem
cal and potentially harmful to patients. from iron deficiency in patients with HF. Most notably, stud-
ies have shown an increase in all-cause mortality in patients
Comorbidity Management in HF
with a concomitant iron deficiency and HF independently of
Anemia anemia (Okonko 2011). In a meta-analysis of five prospective
Beyond optimizing GDMT, comorbidity management can studies, intravenous iron therapy was evaluated for its effect
help improve quality of life and avoid complications. Certain on death and hospitalizations in patients with heart failure

PSAP 2019 BOOK 1 • Cardiology 91 Heart Failure


with reduced ejection fraction (HFrEF) (Jankowska 2016). their negative inotropic activity, which can worsen HF symp-
The study showed that intravenous iron in patients with iron toms (discussed later). For patients with heart failure with
deficiency (regardless of concomitant anemia) improves out- preserved ejection fraction (HFpEF), optimizing diuretic ther-
comes (all-cause death or CV hospitalizations), alleviates HF apy to ensure euvolemia is important before considering
symptoms, and improves overall quality of life. However, this additional treatments. Recently, spironolactone reduced hos-
analysis showed no discernible differences regarding all- pitalizations in patients with HFpEF and should be consid-
cause mortality or CV deaths. Although previous studies have ered in patients with hypertension with this subset of HF (Pitt
been underpowered to detect a survival benefit with iron ther- 2014). Beyond the potential role for spironolactone, data are
apy, the currently enrolling AFFIRM-AHF (left ventricular ejec- sparse to support pharmacotherapy with respect to clinical
tion fraction [LVEF] less than 50%) and IRONMEN (LVEF less outcomes in patients with HFpEF; implementation of therapy
than 45%) trials should provide information on the potential should be driven by comorbidities and tolerability in combina-
benefits of iron therapy for CV death and hospitalizations in tion with patient and provider preferences.
patients with HF (clinicaltrials.gov).
Erythropoiesis-stimulating agents (ESAs) have also been Diabetes
evaluated in patients with HF and anemia. Evidence support- Diabetes is one of the most common comorbidities in patients
ing ESA use has been mixed, with limited sample sizes among with HF and is as a major risk factor for developing new-onset
clinical trials. The largest study to date was a randomized dou- HF. Although caution should be used when selecting agents,
ble-blind study evaluating darbepoetin alfa (n=2278) in symp- certain diabetes medications may help prevent HF and play
tomatic HF (EF of 40% or less with Hgb 9–12.5 g/dL) (Ghali a pivotal role in reducing CV events. In patients with estab-
2008). After 27 weeks, darbepoetin was not associated with lished CV disease, empagliflozin (a sodium-glucose cotrans-
clinical benefits; however, it significantly increased thrombo- porter 2 [SGLT-2] inhibitor) modestly reduced the composite
embolic complications and nonsignificantly increased the outcome (14%, p=0.04 for superiority) of CV mortality, nonfa-
rates of fatal and nonfatal strokes. Despite a meta-analysis tal myocardial infarction (MI), and nonfatal stroke within the
(11 studies) showing the benefits of ESAs (i.e., 6-minute walk, EMPA-REG OUTCOME study (Zinman 2015) when compared
exercise duration, NYHA functional status), the potential to placebo. Findings were largely driven by a 38% reduction
thromboembolic complications associated with this therapy in CV death, but the authors also noted a 32% reduction in
have led the guidelines to recommend avoiding ESA therapies all-cause mortality. In addition, HF hospitalizations were sig-
in patients with HF and anemia. nificantly reduced, but this should be assessed cautiously
because the study did not set rigid criteria to capture the
Hypertension HF type or cardiac function at baseline. Patients within the
Hypertension is an important risk factor for developing HF, study were noted to have an increase in genitourinary infec-
and appropriate therapies should be implemented to avoid tions, which resolved with antifungal therapy. Canagliflozin,
onset. Given the findings of the SPRINT study, the ACC/AHA another SGLT-2 inhibitor, was evaluated within the CANVAS
HF guidelines advocated tighter blood pressure control in program for its potential usefulness in patients at high risk
patients at risk of HF (stage A) (Yancy 2017). Specifically, of CV and diabetes mellitus. Compared with placebo, cana-
patients older than 75 and those with established vascu- gliflozin significantly reduced the risk of the primary compos-
lar disease, chronic renal disease, or a Framingham score ite outcome of death from CV cause, nonfatal MI, or nonfatal
greater than 15% were at a greater risk of HF onset if sys- stroke (26.9 vs. 31.5 participants per 1000 patient-years; HR
tolic blood pressure was greater than 120 mm Hg (SPRINT 0.86; 95% CI, 0.75–0.97; p<0.001 for noninferiority; p=0.02
Research Group 2015). A recent cohort study showed that for superiority) (Neal 2017). Despite a reduction in the pri-
higher baseline diastolic, systolic, and pulse pressures lead mary composite end point, canagliflozin was not able to rep-
to increased adverse events, further emphasizing the impor- licate a reduction in CV mortality that was found within the
tance of blood pressure control within this high-risk popula- EMPA-REG OUTCOME study. However, despite the observed
tion (Lip 2015). Patients whose disease progresses to stage benefit of canagliflozin, its use was complicated by a sig-
C HFrEF with ongoing hypertension should ideally optimize nificant increase in lower-limb (mainly toe) amputation (6.3
GDMT, which includes maximizing the doses of RAAS block- vs. 3.4 participants per 1000 patient-years; HR 1.97; 95% CI,
ers. Alternatively, transitioning to carvedilol from metoprolol 1.41–2.75; p<0.001). Patients with a history of amputations
succinate can be considered with the titration of mineralo- or peripheral vascular disease were at highest risk of ampu-
corticoid receptor antagonists (MRAs) and diuretic therapy. tation within the canagliflozin cohort. A closer evaluation of
In patients who remain hypertensive, adding hydralazine and/ patients randomized within the EMPA-REG OUTCOME and
or dihydropyridine calcium channel blockers (CCBs) (e.g., CAVAS studies can potentially explain why empagliflozin
amlodipine, felodipine) should be considered because these showed a more demonstrable effect on the primary end
agents are all safe in HFrEF. Non-dihydropyridine CCBs (e.g., point. All patients randomized within the EMPA-REG OUT-
verapamil or diltiazem) should be avoided in HFrEF, given COME study had a prior history of CV disease (secondary

PSAP 2019 BOOK 1 • Cardiology 92 Heart Failure


prevention) while only 65% of patients within the CANVAS with placebo (3.5% vs. 2.8%; p=0.007). Although initial find-
study had prior CV disease (both primary and secondary pre- ings did not show an increase in HF hospitalizations in
vention). A difference in patient population could help explain patients randomized to alogliptin, post hoc analyses sug-
the significant reduction within the EMPA-REG OUTCOME gested that patients with no history of HF at study inclusion
study which was not seen in the CANVAS study due the inclu- had increased hospitalizations because of HF (HR 1.76; 95%
sion of an arguably “healthier” study population. Similar to CI, 1.07–2.90; p=0.026). This CV complication did not occur
the CANVAS study, danagliflozin was evaluated in patients with sitagliptin, suggesting that HF hospitalizations is not a
who had or were at risk for atherosclerotic CV disease within class effect.
the DECLARE-TIMI 58 study (Wiviott 2018). Unlike the previ- Metformin continues to be used cautiously in patients with
ously studied medication within this drug class, danagliflozin HF, especially in those with NYHA class III–IV symptoms,
did not show a reduction in the primary composite outcome because this agent has been associated with an increased
of CV mortality, nonfatal MI or nonfatal stroke (8.8% vs 9.4%; risk of lactic acidosis in patients with renal or hepatic impair-
HR 0.93; 95% CI, 0.84-1.03; p=0.17). Given the findings of the ment. Although careful monitoring of kidney and liver func-
EMPA-REG OUTCOME study with respect to reduction in CV tion is warranted with metformin, data analyses support its
mortality and reduction in hospitalization, the oversight com- use in this patient population because it modestly reduced
mittee amended the protocol to include an additional primary morbidity and mortality. Despite the initial report that 47
composite end point, CV death or hospitalization for HF. With patients developed lactic acidosis in association with met-
respect to the added primary outcome, danagliflozin demon- formin use (38% having concomitant HF diagnosis), several
strated a significant benefit compared to placebo (4.9% vs evaluations have failed to replicate this association. In a
5.8%; HR0.83; 95% CI, 0.73-0.95; p=0.005), which was largely large observational study (n=2952) evaluating diabetes ther-
attributed to a reduction in HF hospitalization. The noted dif- apies in patients with HF, no associated hospitalizations or
ferences in outcomes amongst the studies evaluating SLT2 deaths were attributed to metformin-associated lactic acido-
inhibitors could reflect a difference in study population as sis (Andersson 2010). Several small case series have shown
DECLARE-TIMI 58 included the lowest rate of established CV a poor correlation between mortality in patients with lactic
disease (40%). Ongoing studies of HF populations are further acidosis receiving metformin and their associated metformin
explaining the potential risk-benefit of SGLT-2 inhibitors and and lactate serum concentrations. Finally, in a literature eval-
will further guide therapy in these complex patients. uation of 47 patients with lactic acidosis, neither metformin
Liraglutide and semaglutide (glucagon-like peptide 1 [GLP- nor lactate concentrations correlated with mortality (Stades
1] agonists) were shown to significantly reduce the compos- 2004). Given the information to date, metformin appears safe
ite primary outcome of death from CV causes, nonfatal MI, or in patients with HF and should be considered, given its abil-
nonfatal stroke (Marso 2016a, Marso 2016b). Liraglutide was ity to significantly reduce macrovascular events and diabe-
evaluated in the LEADER trial in patients with CV disease or tes-related mortality.
patients with one or more CV risk factors. The primary out-
come was largely driven by a significant reduction in CV mor-
RENIN-ANGIOTENSIN-
tality (p=0.007). Semaglutide was evaluated in the SUSTAIN
ALDOSTERONE SYSTEM
study in a similar patient population, where it also signifi-
cantly reduced the primary composite of CV death, nonfatal Angiotensin II (AngII) is a key hormone within the RAAS path-
MI, and nonfatal stroke (HR 0.74; 95% CI 0.58-0.95; p<0.001 for way that can worsen HF through arterial vasoconstriction,
noninferiority). Unlike liraglutide, the primary end point was stimulation of sodium and water reabsorption, promotion
not driven by a reduction in CV mortality, but nonfatal stroke of cellular proliferation, and evocation of an inflammatory
was significantly improved with a trend toward a lower inci- response. Traditional therapies designed to blunt these effects
dence of MI. It is unclear whether the overall CV outcomes include angiotensin-converting enzyme inhibitors (ACEIs),
with GLP-1 agonists are a class effect or whether certain CV angiotensin receptor blockers (ARBs), and MRAs.
benefits are provided by individual medications.
Dipeptidyl peptidase 4 (DPP-4) inhibitors have been scru- ACE Inhibitors
tinized, given their variable benefits on CV outcomes and Angiotensin-converting enzyme inhibitors form the foun-
safety profile. To further understand the concern surrounding dation for HFrEF management and are recommended as
safety of DPP-4 inhibitor use in HF, three CV safety studies first-line therapy, given their ability to reduce morbidity and
examined saxagliptin (SAVOR-TIMI 53), sitagliptin (TECOS), mortality. Therapy with ACEIs should be initiated according to
and alogliptin (EXAMINE) (Green 2015; Scirica 2013; White the starting doses used in clinical trials, with a goal of titrating
2013). These agents were not associated with reduced CV to the target doses that showed mortality benefit (Table 2).
outcomes, contrary to results from a previous meta-analy- The beneficial outcomes of ACEI therapy are considered a
sis. Despite meeting the safety information requested by the class effect, unlike other drug classes used in HF therapy.
FDA, saxagliptin increased HF hospitalizations compared

PSAP 2019 BOOK 1 • Cardiology 93 Heart Failure


Table 2. Approved HF Medications

Drug Starting Dose Target Dose Mean Dose Achieved in Clinical Trial

ACEI (Class I; LOE A)

Captopril 6.25 mg TID 50 mg TID 122.7 mg total daily dose

Enalapril 2.5 mg BID 10–20 mg BID 16.6 mg total daily dose

Lisinopril 2.5–5 mg QD 20–40 mg QD 32.5–35 mg total daily dose

ARBs (Class I; LOE A)

Candesartan 4–8 mg QD 32 mg QD 24 mg total daily dose

Losartan 25–50 mg QD 50–150 mg QD 129 mg total daily dose

Valsartan 20–40 mg BID 160 mg BID 254 mg total daily dose

ARNI (Class I; LOE B)

Sacubitril/valsartan 49/51 mg BID 97/103 mg BID 375 mg total daily dosea

MRAs (Class I; LOE A)

Spironolactone 12.5–25 mg QD 25 mg QD or BID 26 mg total daily dose

Eplerenone 25 mg QD 50 mg QD 42.6 mg total daily dose

β-Blocker (Class I; LOE A)

Bisoprolol 1.25 mg QD 10 mg QD 8.6 mg total daily dose

Carvedilol 3.125 mg BID 25 mg BID 37 mg total daily dose


50 mg BID (if ≥ 85 kg)

Metoprolol succinate 12.5–25 mg QD 200 mg QD 159 mg total daily dose

Isosorbide dinitrate/hydralazine (Class I; LOE A) b

Fixed-dose combination 20 mg of isosorbide 40 mg of isosorbide 90 mg of isosorbide dinitrate/~175 mg


dinitrate/37.5 mg of dinitrate/75 mg of of hydralazine QD
hydralazine TID hydralazine TID

Isosorbide dinitrate and 20–30 mg of isosorbide 40 mg of isosorbide Not studied


hydralazine dinitrate/25–50 mg of dinitrate TID with 100 mg
hydralazine TID or QD of hydralazine TID

If channel inhibitor (Class IIa; LOE B)

Ivabradine 5 mg BID 7.5 mg BID 6.4 mg BID (28 days) and 6.5 mg BID
(1 year)

a
Total dose of both components as defined in the PARADIGM-HF study.
b
African American patients who remain symptomatic despite receiving GDMT.
ACEI = angiotensin-converting enzyme inhibitor; ARB = angiotensin receptor blocker; ARNI = angiotensin receptor-neprilysin
inhibitor; BID = twice daily; GDMT = guideline-directed management and therapy; HF = heard failure; LOE = level of evidence; MRA =
mineralocorticoid receptor antagonist; QD = once daily; QID = four times daily; TID = three times daily.
Information from: Yancy CW, Jessup M, Bozkurt B, et al. 2017 ACC/AHA/HFSA focused update of the 2013 ACCF/AHA guideline for
the management of heart failure: a report of the American College of Cardiology/American Heart Association Task Force on Clinical
Practice Guidelines and the Heart Failure Society of America. Circulation 2017;136:e137-61.

The adverse effect profile of ACEIs is primarily reflected rare, complication of ACEI therapy is angioedema. Within the
in direct or indirect inhibition of AngII formation or because ONTARGET clinical trial, angioedema occurred in only 0.3%
of increased bradykinin concentrations (Table 3). Common of 8500 patients treated with ramipril (ONTARGET Investiga-
adverse events include hypotension, hyperkalemia, worsen- tors 2008). Patients who develop angioedema while receiv-
ing renal function, and dry cough. The most severe, albeit ing ACEI therapy should avoid being rechallenged, given the

PSAP 2019 BOOK 1 • Cardiology 94 Heart Failure


Table 3. Traditional HF Therapies: Mechanism of Action, Adverse Effects, and Monitoring

Compensatory
Mechanism
Affected Drug Class Mechanism of Action Adverse Effects (common) Monitoring Values

RAAS ACEIs Competitive inhibitor of ACE • Hypotension • K


leading to a reduction in the • Hyperkalemia • SCr
conversion of AngI to AngII • Worsening renal function • BP
• Dry cough
• Angioedema (rare)

ARBs Inhibition of the AngII type 1 • Hypotension • K


receptor • Hyperkalemia • SCr
• Worsening renal function • BP

MRAs Inhibition of the aldosterone • Hyperkalemia • K


receptor • Worsening renal function • SCr
• Gynecomastia
(spironolactone)

Sympathetic β-Blockers Inhibition of β-adrenoreceptor • Bradycardia • Heart rate


nervous system and potentially α-adrenergic • Hypotension • BP
activity • Dizziness

Nitrates Increased cGMP concentrations • Headache • BP


leading to smooth muscle • Dizziness
relaxation • Hypotension

Hydralazine Direct vasodilation of arterioles • Headache • BP


• Dizziness
• Hypotension

AngII = angiotensin II; BP = blood pressure; cGMP = guanosine 3’5’-monophosphate; HF = heart failure; RAAS = renin-angiotensin-al-
dosterone system.

concern for recurrence. To limit the risk of adverse effects, trials. Monitoring should occur after initiating therapy and
ACEI therapy should be titrated after at least 2 weeks. Clinical after dose escalations.
laboratory values and adverse effects should be monitored
after each titration. Aldosterone Receptor Antagonist Therapy
Mineralocorticoid receptor antagonists have been used
ARB Therapy across the HF spectrum, providing benefits through inhibit-
Angiotensin receptor blockers were developed with the idea ing aldosterone’s activities. Clinical studies have shown sur-
that inhibiting AngII through blockade of the AngII type 1 vival benefits when MRAs are used in patients with advanced
receptor could further improve HF. The current guidelines HF (spironolactone) and patients after an MI with a reduced
recommend that either an ACEI or an ARB be used, with EF (eplerenone). Use of MRAs in patients with mild-moderate
ARBs preferred if patients cannot tolerate an ACEI. Unlike HF was studied in the EMPHASIS-HF study. Patients were
ACEI therapy, whose benefits are considered universal well matched at randomization, with 93% of study partici-
across the drug class, the following ARBs are listed for use pants taking an ACEI or an ARB (or both) and 86% receiving
within the current guidelines: candesartan, valsartan, and β-blockers. Like within the RALES study, the overwhelm-
losartan. ing benefit of eplerenone in addition to GDMT prompted
Angiotensin receptor blockers are generally well toler- the study to be prematurely terminated because of a 37%
ated and have an adverse effect profile similar to ACEIs, with reduction in the composite primary end point: risk of death
a noted reduction in the rates of cough and angioedema and hospitalization for HF compared with placebo (Zannad
but an apparent increased rate of hypotension. Titration of 2011). Patients were initiated on eplerenone 25 mg/day with
ARBs should be similar to that of ACEIs, with initiation at low a dose increase at 4 weeks to a goal dose of 50 mg/day (or in
doses and increasing doses similar to those used in clinical patients with an estimated glomerular filtration rate [eGFR]
of 30–49 mL/minute/1.73 m2, study investigators initiated

PSAP 2019 BOOK 1 • Cardiology 95 Heart Failure


25 mg every other day with an increase to 25 mg daily at Given the overt risk of hyperkalemia with MRAs, careful
4 weeks) if serum potassium concentrations remained monitoring of potassium concentrations and renal function
below 5 mEq/L. After 5 months of therapy, the mean doses is warranted when initiating therapy and particularly within
achieved in the eplerenone and placebo groups were 39.1 the first few months, or when doses are adjusted.
mg (±13.8) and 40.8 mg (±12.9), respectively.
Because of findings from the EMPHASIS-HF trial, the SYMPATHETIC ACTIVATION
guidelines state that MRAs should be implemented in In patients with HF having NYHA class II–IV symptoms,
patients with NYHA classes II–IV receiving GDMT with a bisoprolol, carvedilol, or metoprolol (controlled release or
CrCl greater than 30 mL/minute/1.73 m2 and a K less than extended release), when combined with ACEI, digoxin, and
5.0 mEq/L. Pharmacists play an essential role in ensuring diuretics, reduced the composite end point of hospital admis-
that MRAs are added to patients’ HF regimens because stud- sion and mortality by over 30%. The current guidelines only
ies have shown a continued underuse of this highly effective recommend metoprolol succinate, bisoprolol, and carve-
medication class. dilol because these are the only three β-blockers proven to
In clinical studies, MRAs were well tolerated, with study reduce mortality; the benefits are not thought to be a class
drug discontinuation rates similar to placebo. Most nota- effect. Data are limited comparing the relative efficacies of
bly, MRAs have consistently significantly increased the risk the currently approved β-blockers for HFrEF. Carvedilol may
of hyperkalemia. Male patients randomized to spironolac- be advantageous in patients with HF having baseline hyper-
tone had a significant increase in the development of gyne- tension versus bisoprolol or metoprolol in patients with mar-
comastia, which did not occur with eplerenone because of ginal blood pressure at initiation.
its different molecular moiety. Caution with MRAs should be In β-blocker–naive individuals, starting doses should
used in patients whose baseline potassium concentrations be low and therapy initiated when patients are clinically
are greater than 5 mEq/L or with impaired renal function compensated, with the goal of titrating to maximally tol-
(SCr greater than 2.5 mg/dL in men or greater than 2 mg/dL erated doses or the target doses used in clinical trials. In
in women or eGFR less than 30 mL/minute/1.73 m2) because patients who present with acute decompensated HF pre-
both characteristics were excluded from the original studies. viously receiving β-blocker therapy, continued therapy is

Patient Care Scenario


A 35-year-old African American man was referred to with a reported EF of 35%. Following guideline recommen-
cardiology for newly diagnosed HF thought to be con- dations, the patient is initiated on enalapril 5 mg twice
genital, given that both of his parents and his older sister daily, metoprolol succinate 25 mg daily, and spironolac-
had previously been given a diagnosis of HF. The patient tone 25 mg daily. Three days after starting therapy, the
denies active concerns but states that he recently has patient begins to have worsening shortness of breath with
felt increased shortness of breath with an inability to lie a swelling sensation of the lips, mouth, and throat. The
flat when he goes to bed at night. He still performs his resident is concerned about ACEI-induced angioedema
activities of daily living without issues. Echocardiography and seeks your recommendation. What is best to recom-
(ECHO) reveals moderately reduced ventricular function mend for this patient?
ANSWER
This unique scenario shows a significant complication monitoring should ensue to ensure patients do not have
resulting from ACEI use. Development of angioedema with cross-reactivity. Transitioning to sacubitril/valsartan
ACEI use should preclude future use within this specific would be ideal, given the mortality benefits associated
drug class because of the significant risk of recurrence. with this new agent; however, because of a high likelihood
The best option for the patient, given the reduced risk of angioedema, its use is contraindicated. Use of alter-
of angioedema associated with ARB therapy, would be a native medication classes (CCBs or hydralazine/nitrates)
transition to a medication within this drug class. Although should be reserved as second-line therapies if a similar
the likelihood is significantly reduced, patients receiving reaction occurs with ARBs.
ARBs have reported episodes of angioedema, and careful

1. C ohn JN, Tognoni G. A randomized trial of the angiotensin-receptor blocker valsartan in chronic heart failure. N Engl J Med
2001;345:1667-75.
2. M cMurray JJ, Packer M, Desai AS, et al. Angiotensin-neprilysin inhibition versus enalapril in heart failure. N Engl J Med 2014;371:993-1004.
3. P feffer MA, McMurray JJ, Velazquez EJ, et al. Valsartan, captopril, or both in myocardial infarction complicated by heart failure, left
ventricular dysfunction, or both. N Engl J Med 2003;349:1893-906.
4. Y ancy CW, Jessup M, Bozkurt B, et al. 2017 ACC/AHA/HFSA focused update of the 2013 ACCF/AHA guideline for the management of heart
failure: a report of the American College of Cardiology/American Heart Association Task Force on Clinical Practice Guidelines and the
Heart Failure Society of America. Circulation 2017;136:e137-61.

PSAP 2019 BOOK 1 • Cardiology 96 Heart Failure


recommended unless hemodynamic compromise is evident NOVEL HF THERAPIES
through requirement of inotropes or mechanical circulatory Angiotensin Receptor-Neprilysin Inhibitors
support.
Natriuretic peptides (NPs) play an important role in HF, serv-
Patients should be monitored closely after initiating (or
ing as potential targets in managing the disease. Histori-
dose titrating) β-blocker therapy to assess for potentially
cally, augmenting NPs with nesiritide (synthetic BNP) did not
worsening symptoms of HF because of the β-blocker’s neg-
have long-term beneficial effects when assessed in patients
ative inotropic properties. At least 2 weeks is recommended
with acute HF (O’Connor 2011). Preventing the breakdown of
between dose titrations, and some patients may require
endogenous NPs as a target led to the development of sacu-
extended titration intervals. Patients who are decompen-
bitril, a neutral endopeptidase that blocks neprilysin, the
sated or hypervolemic at initiation are at a higher risk of
enzyme responsible for degrading NPs. Inhibition of nepri-
worsening HF. Bradycardia, hypotension, dizziness, and
lysin may lead to improved fluid homeostasis and reduced
bronchospasms are additional adverse effects that should
neurohormonal activation. Neprilysin is also responsible for
be closely monitored after β-blocker initiation and subse-
degrading several other vasoactive peptides, including AngII.
quent dose titrations. Patients who develop symptomatic
Thus, inhibiting neprilysin alone results in increased con-
hypotension while taking carvedilol should be changed to
centrations of both natriuretic peptides and AngII. As such,
metoprolol succinate or bisoprolol. Alternatively, a dose
neprilysin must be combined with a RAAS blocker to negate
reduction can be considered.
elevated AngII concentrations. Combining valsartan (an ARB)
with a neprilysin inhibitor prodrug sacubitril created sacubi-
ALTERNATIVE AGENTS
tril/valsartan (formerly LCZ 696 [Entresto]), the first ARNI in
Nitrates/Hydralazine its class.
Hydralazine plus oral nitrates is generally reserved for The PARADIGM-HF study was a phase III randomized trial
patients who have a contraindication to or cannot tolerate including 8399 patients with NYHA class II–IV HF symptoms,
first-line agents (ACEIs, ARBs, angiotensin receptor-nepri- EF of 40% or less (changed to 35% or less 1 year into enroll-
lysin inhibitors [ARNIs]) or in the setting of ongoing NYHA ment), and elevated NT-proBNP concentrations receiving
class III–IV HF symptoms despite receiving optimal GDMT GDMT for HF. The investigators hypothesized that sacubitril/
in patients of African American descent. valsartan would better reduce morbidity and mortality than
Adherence should be a factor when considering adding enalapril (goal dose 20 mg/day) (McMurray 2014). Interim
hydralazine/nitrates to a patient’s pharmacotherapy plan. safety and efficacy data analyses showed that sacubitril/val-
Given their short half-lives, these drugs are given three sartan resulted in a 20% relative risk reduction in the com-
times daily. As with other dose titrations, patients should posite end point (CV death and hospitalization for HF) as well
be evaluated for at least 2 weeks before increasing the as a 16% relative risk reduction in all-cause mortality, and
doses to target doses. Long-acting once-daily isosorbide the study was terminated early because of the overwhelm-
mononitrate (40–120 mg daily) has been suggested as an ing efficacy of sacubitril/valsartan. Because of the results
alternative to the dinitrate formulation to overcome adher- of the PARADIGM-HF study, the HF guidelines recommend
ence concerns, but direct evidence is lacking. Even though that in patients currently tolerating ACEI or ARB therapy,
the fixed-dose combination of hydralazine/nitrate was ben- sacubitril/valsartan replacement should be used to further
eficial in clinical studies, its overall cost has prompted cli- reduce morbidity and mortality. The guidelines do not cur-
nicians to individually dose hydralazine and isosorbide rently recommend sacubitril/valsartan in patients with de
dinitrate, even though this will increase overall pill burden. novo HF because this population was not included in PAR-
Common adverse effects of hydralazine/nitrates include ADIGM-HF, but the recently published PIONEER study evalu-
headache, dizziness, and hypotension. A significantly ated the safety and efficacy of sacubitril/valsartan in patients
greater risk of headaches (48% vs. 19%) and dizziness was hospitalized for acute decompensated HF (Velazquez 2018).
noted in 29% of patients (placebo 12%) within the A-HeFT Once hemodynamically stable patients were randomization
trial (Taylor 2007). If symptomatic hypotension occurs, to receive enalapril (initial dose: 2.5 mg or 5 mg twice daily;
reduction in hydralazine/nitrate is advocated, but concur- target dose: 10 mg twice daily) or sacubitril/valsartan (initial
rent first-line therapies (β-blocker, ACEI, or ARB) should be dose: 24 mg of sacubitril with 26 mg of valsartan or 49 mg of
continued, if possible. In rare cases, patients may develop sacubitril with 51 mg of valsartan twice daily; target dose 97
drug-induced lupus syndrome (hydralazine) or vasculitis mg of sacubitril with 103 mg of valsartan twice daily). The pri-
(noted by hematuria, SCr elevation, and edema). If either mary efficacy outcome was the time-averaged proportional
syndrome develops, immediate evaluation is warranted, in change in NT-proBNP from baseline through weeks 4 and 8.
addition to prompt discontinuation of hydralazine/nitrate. A significant reduction in the NT-proBNP concentration were
seen amongst patients randomized to sacubitril/valsartan
(n=440) as opposed to enalapril (n=441) (percent change,

PSAP 2019 BOOK 1 • Cardiology 97 Heart Failure


-46.7% vs -25.3%; ratio of change with sacubitril/valsartan vs those randomized to enalapril had a higher rate of severe
enalapril, 0.71; 95% CI, 0.63-0.81; p<0.001). Safety outcomes hyperkalemia (greater than 6 mEq/L) (3.1 vs. 2.2 per 100
which included risk for worsening real function, hyperkalemia patient-years; p=0.02) than did those randomized to sacubi-
development, and hypotension, were not different amongst tril/valsartan (Desai 2016). Patients naive to MRAs at base-
the study groups. It is important to note the benefits demon- line of the PARADIGM-HF study had similar rates of severe
strated with sacubitril/valsartan were seen in patients irre- hyperkalemia, irrespective of randomization group. Because
spective of history of HF or previous use of ACEi or ARB, concerns for hyperkalemia often limit the use of MRAs in
suggesting that the use of sacubitril/valsartan can likely be chronic HF, the lower risk of hyperkalemia with sacubitril/val-
utilized safely and effectively in de novo HF patients. A sub- sartan may be reassuring to providers.
sequent analysis of the PARADIGM data showed that sacu- Patients new to ARNI therapy or previously taking low-
bitril/valsartan was associated with an incremental benefit dose ACEI therapy (less than 10 mg/day of enalapril or equiv-
over enalapril on the nonfatal progression of HF (intensifica- alent) or ARB therapy (less than 160 mg/day of valsartan or
tion of therapy [HR 0.84; 95% CI, 0.74–0.94; p=0.003]) (Packer equivalent) should be initiated on sacubitril/valsartan 24/26
2015). Sacubitril/valsartan significantly reduced the risk of a mg twice daily. The recommended starting dose for patients
first ED visit (HR 0.66; 95% CI, 0.52–0.85; p=0.001) or hospital- receiving moderate- to high-dose ACEI or ARB therapy is
ization (HR 0.79; 95% CI, 0.52–0.94); p=0.017), in addition to 49/51 mg twice daily. Patients with moderate hepatic impair-
decreasing the need for repeated visits for HF compared with ment (Child-Pugh class B) or severe renal impairment (eGFR
enalapril. Moreover, a noted intensification of background less than 30 mL/minute/1.73 m2) should also be considered
therapies within the enalapril group testified to improvements for a starting low dose of sacubitril/valsartan 24/26 mg twice
in HF with sacubitril/valsartan. The benefits with sacubitril/ daily. Patients should be monitored for 2–4 weeks before
valsartan were noted as early as 30 days after randomization, doubling the dose to the target maintenance dose of 97/103
further highlighting the importance of this new medication. mg twice daily. Patients should be monitored for hypoten-
Despite the overwhelming benefit shown within the PAR- sion, worsening renal function, and hyperkalemia after dose
ADIGM-HF study, questions surrounding its study design adjustments and periodically while receiving treatment. Of
and conclusions have emerged. A run-in period was used importance, as shown in the PARADIGM-HF study, BNP con-
to establish the tolerability of the study drug, but this tech- centrations increase while patients are receiving sacubitril/
nique may create results that represent a subset of a larger valsartan therapy because of reduced BNP degradation by
patient population. To better quantify the generalizability of neprilysin (McMurray 2014). However, given that NT-proBNP
the PARADIGM-HF results, an analysis was performed using is not a substrate for the neprilysin enzyme, it is appropriate
patient characteristics from pre-randomization exclusions for use as a marker of HF severity while sacubitril/valsartan
for the treatment effect of the primary composite end point therapy is used. In PARADIGM-HF, higher BNP concentrations
(death from CV causes or first hospitalization for HF) (Desai but lower NT-proBNP concentrations were noted at 4 weeks
2016). A total of 2079 patients (19.8%) did not complete the and 8 months compared with placebo group (Packer 2015).
run-in phase, with most unable to be randomized because of Before initiating sacubitril/valsartan and when interchang-
adverse events (12%) or withdrawal of consent (2.6%). Reanal- ing sacubitril/valsartan and an ACEI, a washout period of 36
ysis using inverse probability weighting confirmed the benefit hours should occur to avoid the development of angioedema.
of sacubitril/valsartan over enalapril. Sacubitril/valsartan is contraindicated in patients with a his-
Overall, sacubitril/valsartan was well tolerated with fewer tory of angioedema.
study medication discontinuations compared with enal-
april (17.8% vs. 19.8%). Within the sacubitril/valsartan group, Ivabradine
patients were at a higher risk of developing symptomatic The premise that heart rate is an important factor associ-
hypotension (14% vs. 9.2%, p<0.001), which did not influence ated with HF mortality led to the development of ivabra-
the need for permanent therapy discontinuation. In addition, dine. Unlike β-blockers, which are often dose limited by
hypotension within the run-in phase did not differentially hypotensive episodes, ivabradine reduces heart rate with-
affect the relative efficacy of sacubitril/valsartan compared out compromising blood pressure. By directly inhibiting the
with enalapril after randomization (Vardeny 2018). Of interest, funny channel (sodium channel) within the sinoatrial node,
these hypotensive effects did not correlate with worsening ivabradine prolongs the slow depolarization phase and sub-
renal function (SCr greater than 2.5 mg/dL: 3.3% sacubitril/ sequently reduces heart rate. Ivabradine failed to show mor-
valsartan vs. 4.5% enalapril, p=0.07). A lower risk of serum tality or hospital admission benefit when originally evaluated
potassium elevations greater than 6 mEq/L (4.3% vs. 5.6%, in chronic stable coronary artery disease in the BEAUTIFUL
p=0.07) and a reduction in cough episodes (11.3% vs. 14.3%, trial; however, a subgroup analysis showed a treatment bene-
p<0.001) occurred with sacubitril/valsartan. A secondary fit in patients with a baseline heart rate greater than 70 beats/
analysis directed at further explaining the risk of hyperkale- minute. The subsequent SHIFT study evaluated ivabradine in
mia showed that among patients receiving MRAs at baseline, 6500 patients with HF (EF of 35% or less), NYHA classes II–IV,

PSAP 2019 BOOK 1 • Cardiology 98 Heart Failure


and a resting heart rate of 70 beats/minute or more (Swed- available safety data showed a 15% increased risk of develop-
berg 2010). After a 23-month follow-up, ivabradine reduced ing AF with ivabradine compared with control, which is asso-
the composite end point (CV death or hospitalization) by 18% ciated with a number needed to harm (NNH) of 208 (Martin
(24% vs. 29%, p<0.0001). These findings were largely driven by 2014). Looking specifically at data from the BEAUTIFUL and
a reduction in hospitalizations (16 vs. 21%; p<0.001). Although SHIFT trials, patients with a baseline heart rate of 70 beats/
no difference occurred in all-cause (16% vs. 17%, p=0.092) or minute or greater had an even higher rate of AF, with an NNH
CV mortality (14% vs. 15%, p=0.128), HF mortality was reduced of 58. Of note, patients at highest risk of AF are also the most
with ivabradine (3% vs. 5%, p=0.014). likely to benefit from ivabradine. To further investigate the
A detailed examination of the SHIFT baseline characteris- association of ivabradine treatment and AF development, a
tics merits further discussion. Although 89% of patients were meta-analysis (n=40,437) was completed using three ran-
reportedly receiving β-blocker therapy, 14% were receiving domized controlled trials and data from the European Med-
an agent that did not follow the current ACC/AHA guideline icines Agency. The incidence of AF was 5.34% in patients
recommendations (e.g., nebivolol, metoprolol tartrate). A key receiving ivabradine and 4.56% in patients receiving placebo,
limitation of the study was related to the dosing of β-blocker correlating with a statistically significant 24% relative risk
therapy. Only 56% of patients received one-half the recom- increase in patients receiving ivabradine therapy (Tanboga
mended target dose, and only 26% received the target dose. 2016). Given these findings, careful consideration should be
In about 75% of patients, the target dose was not achieved given to initiating ivabradine therapy, and this agent should
because of hypotension or fatigue. Sub-target β-blocker use be considered contraindicated in patients with AF.
raises questions about the benefit of ivabradine in patients In a certain subset of patients with HF, despite maximum
receiving optimal GDMT. Furthermore, the profound mortality GDMT, the severity of symptoms will become recurrent and
benefits of β-blockers in several clinical trials warrant more persistent. Patients who require an increasing level of care
aggressive measures to reach target doses before prescrib- and treatment intensity with the noticeable failure of oral
ing ivabradine, which has not yet shown a mortality benefit. therapies have been categorized as patients with advanced
Given the many benefits with β-blocker therapy, the cur- HF (Allen 2012). Patients whose disease progresses to stage
rent guidelines recommend that ivabradine be considered D HF should be referred to centers that specialize in advanced
in patients with stable HFrEF in sinus rhythm and a resting therapies, including inotropic therapy, left ventricular assist
heart rate of 70 beats/minute or more who are receiving max- devices (LVADs), and heart transplantation.
imum tolerated β-blocker therapy or who have a contraindi-
cation to β-blocker use. Patients should be initiated on 5 mg INOTROPIC THERAPY
twice daily; however, an initial dose of 2.5 mg twice daily may Inotropic therapy can significantly increase the risk of mor-
be considered in patients for whom bradycardia could lead tality. Given that studies showing worse outcomes were con-
to hemodynamic compromise. After initiating therapy, in ducted when implantable internal cardiac defibrillator or
patients who remain asymptomatic with a heart rate greater chronic resynchronization therapy (ICD/CRT) devices and rou-
than 60 beats/minute, titration to a target dose of 7.5 mg tine β-blocker therapy were not in common practice, inotropic
twice daily is recommended. If not receiving maximum ther- therapy has resurfaced as a potential therapeutic modality in
apy and heart rate is 50–60 beats/minute, patients should be patients with advanced HF. Patients with systolic dysfunc-
maintained on the current dose. If the heart rate is less than tion, hemodynamic compromise including low blood pres-
50 beats/minute or the patient has signs or symptoms con- sure, impaired perfusion, and significantly depressed cardiac
sistent with bradycardia, a reduction to 2.5 mg twice daily is output should be considered for inotropic therapy. Patients
recommended. presenting in cardiogenic shock should be considered for
Symptomatic bradycardia and visual effects were more inotropic therapy, with a definitive therapy (coronary revascu-
common in the ivabradine group (5% vs. 1%, and 3% vs. 1%, larization, mechanical circulatory support, or heart transplan-
respectively) within the SHIFT study. Patients taking ivabra- tation) as the ultimate treatment goal. Long-term continuous
dine developed luminous phenomena or sensations of infusion therapy in patients whose condition is refractory
enhanced brightness, which likely resulted from inhibition to GDMT can be used as a “bridge” strategy while awaiting
of the Ih ion channel in the retina. These symptoms typically permanent therapy (LVAD, transplantation). In advanced HF
presented an average of 40 days after starting ivabradine patients receiving ambulatory intravenous inotropic therapy,
and were considered mild, transient, and fully reversible on a recent systemic review and meta-analysis demonstrated an
discontinuation. The risk of developing atrial fibrillation (AF) improvement in NYHA functional without a significant differ-
within the ivabradine cohort was 9%, with 4% requiring with- ence in the risk of mortality. While the data within the analysis
drawal from the study because of this complication. After were too limited to be stratified by indication (e.g., palliation,
publication of the SHIFT trial, a growing concern surround- bridge to transplant), there appears to be a role for patients
ing the underreporting of AF development when initiating who have progressed to stage D HF without the previously
ivabradine began to surface. In 2014, a meta-analysis using conceived increase in mortality (Nizamic 2018).

PSAP 2019 BOOK 1 • Cardiology 99 Heart Failure


Dobutamine and milrinone remain the most commonly of at least 150 milliseconds and left bundle branch block pat-
used inotropic therapies within this setting. Dobutamine, tern are most likely to improve with CRT therapy (Stavrakis
a β-agonist, and milrinone, a phosphodiesterase inhibitor, 2012). When CRT therapy was extended to include patients
increase cardiac contractility, which subsequently increases with milder HF (NYHA class I–II) symptoms, the results indi-
cardiac output. cated a reversing or halting in LV remodeling in addition to a
Use of inotropic agents can be limited by the ensuing reduction in hospitalizations (Moss 2009).
adverse effects. Because both dobutamine and milrinone
alter cardiac contractility, arrhythmias often develop, with LVAD and Transplantation
rates as high as 50%, necessitating careful monitoring. Patients with advanced HF (stage D) should be considered
Furthermore, peripheral activation of β-receptors and inhi- for advanced therapies, including LVAD implantation or car-
bition of phosphodiesterase can lead to vasodilation, which diac transplantation. Cardiac transplantation remains the
can in turn lead to extreme hypotension. Milrinone is elimi- only definitive treatment for patients with HF unless signif-
nated renally and should be carefully monitored in patients icant contraindications exist (Figure 1). However, the num-
with developing renal dysfunction, given its normal half-life ber of available organ donors does not match the number of
of 2.5 hours. potential needed recipients. Development of long-term LVAD
therapy has provided an alternative therapeutic modality for
patients whose disease is refractory to GDMT and ICD/CRT
ADVANCED THERAPY MODALITIES
devices. Improved technology and treatment of patients with
ICD/CRT Devices LVADs has ensured that patients can successfully be bridged
As part of the compensatory mechanisms in HF, patients to transplantation or be maintained on these devices as des-
often develop hypertrophy of the myocardium in an effort to tination therapy. Patients who have relative or absolute con-
increase cardiac output. Alterations within the cardiac mus- traindications to transplantation are considered destination
cles can predispose patients to ventricular tachyarrhythmias therapy candidates, whereas patients who are eligible for
and, ultimately, sudden cardiac death (SCD). Neurohormonal transplantation can be bridged with an LVAD until a suitable
antagonists have helped reverse cardiac remodeling and the donor is available.
risk of arrhythmias; however, patients with HF are still at a
significant risk of SCD. Several randomized controlled trials
CURRENT HF MEDICATIONS
have evaluated ICD therapy for primary prevention, showing
a reduction in all-cause mortality of 23%–31% (Bardy 2005). The dramatic benefit of using pharmacotherapy in patients
Smaller trials have replicated these findings, except in patients with HFrEF has not been replicated in patients with HFpEF.
with a recent MI (less than 40 days previously), in which no Because of the limited results within HFpEF, most recommen-
survival benefit was shown. The current guidelines recom- dations focus on management strategies to alleviate symp-
mend that primary prevention be considered in patients with toms and treat concomitant conditions.
an LVEF of less than 35% while receiving optimal GDMT for at
least 3–6 months. Because GDMT has potentially improved Aldosterone Receptor Antagonists
EF after therapy initiation, repeat ventricular function assess- The TOPCAT study evaluated the MRA spironolactone in
ment is important to ensure that ICD implantation is still war- patients with symptomatic HF and an LVEF greater than 45%
ranted. Use of ICD therapy is not without consequences, and with respect to the composite primary end point of death from
shocks may be inappropriate (because of the presence of CV causes, aborted cardiac arrest, or hospitalization for HF
artifact or AF) or appropriate but unnecessary (self-terminat- (Pitt 2014). The study randomized 3445 patients to receive
ing ventricular tachycardia), which can lead to reduced qual- placebo or spironolactone. Although the primary outcome
ity of life or posttraumatic stress syndrome (Sears 2011). In rates did not differ (18.6% vs. 20.4%, p=0.14), spironolactone
addition, in rare cases, patients may develop complications reduced HF hospitalization rates (12% vs. 14.2%, p=0.04) after
from the implant surgery, including hematomas or infections. a mean follow-up of 3.3 years. However, in a post hoc analysis,
In a certain subset of patients with HF (30%), progression regional variations potentially skewed the study results (Pfef-
of HF can lead to a prolongation of the QRS interval, which fer 2015). Study patients in Russia/Georgia were younger and
can in turn lead to worse outcomes. Use of CRT or biventricu- had a lesser incidence of AF/diabetes mellitus, a higher like-
lar pacing reverses ventricular remodeling and improves ven- lihood of MI or hospitalization, and a lower baseline EF. The
tricular contraction. The initial studies evaluating the use of analysis showed a significant difference in primary compos-
CRT therapy were in patients with NYHA class III or ambu- ite event rates for the spironolactone and placebo groups in
latory class IV symptoms with a QRS greater than 120–130 the Americas (10.4 and 12.6 per 100 patient-years) and Rus-
milliseconds. The results showed a reduction in rehospital- sia/Georgia (2.5 and 2.3 per 100 patient-years) (p<0.001 for
ization as well as all-cause mortality of 24%–36% (Cleland comparison of rates between regions). Given the large vari-
2005). Increasing evidence suggests that patients with a QRS ance in event rates amongst study centers, an assessment of

PSAP 2019 BOOK 1 • Cardiology 100 Heart Failure


Figure 1. Heart failure treatment algorithm.
AA = African American; ARNI = angiotensin receptor-neprilysin inhibitor; BB = β-blocker; BNP = B-type natriuretic peptide; GDMT =
guideline-directed management and therapy; HFpEF = heart failure with preserved ejection fraction; HFrEF = heart failure with
reduced ejection fraction; ICD/CRT = internal cardiac defibrillator or chronic resynchronization therapy; LVAD = left ventricular
assist device; MI = myocardial infarction; MRA = mineralocorticoid receptor antagonist.
Information from: Yancy CW, Jessup M, Bozkurt B, et al. 2017 ACC/AHA/HFSA focused update of the 2013 ACCF/AHA guideline for
the management of heart failure: a report of the American College of Cardiology/American Heart Association Task Force on Clinical
Practice Guidelines and the Heart Failure Society of America. Circulation 2017;136:e137-61.

PSAP 2019 BOOK 1 • Cardiology 101 Heart Failure


canrenone concentrations (an active metabolite of spirono- preload-dependent, leading to underfilling of the LV and
lactone) was completed in 366 patients (206 from the Ameri- potential hypotension if water removal occurs too rapidly.
cas and 160 from Russia) randomized within the TOPCAT Trial
(de Denus 2017). Despite higher rates of reported medication ACEI/ARB Therapy
continuation at the 12-month visit (Russia: 94.3% vs Ameri- Despite evidence within HFrEF for ACEI/ARB therapy, evi-
cas: 75.2%), a significantly higher proportion of participants dence is lacking for use of this therapy for morbidity or
from Russia had undetectable canrenone concentrations as mortality benefits in patients with HFpEF. In the CHARM-Pre-
compared to Americas participants (30% vs 3%, p<0.001). served study (n=3023), patients randomized to candesartan
These findings, coupled with the a higher incidence of hyper- had significantly fewer HF hospitalizations (p=0.017) and a
kalemia and creatinine changes in the Americas (post hoc trend toward reduction in the primary composite end point
analysis) raises concerns regarding adherence with study (HF hospitalization and cardiac death) (Yusuf 2003). How-
medication at some sites in Russia. While further studies are ever, when irbesartan was evaluated in HFpEF (I-PRESERVE),
warranted to evaluate potential regional differences noted it had no noted benefits over placebo for any of the measured
within the post hoc analysis and subsequent concentration outcomes after almost 50 months of therapy (Massie 2008).
analysis, the TOPCAT study demonstrated a reduction in hos- Given the potential benefits with candesartan, ARBs may be
pital admissions in patients randomized to spironolactone. considered to help reduce HF hospitalizations.
Given the study’s results, the current guidelines recommend
that patients with HF with an EF greater than 45%, an elevated ARNI Therapy
BNP, or hospitalization within 1 year should be considered for The PARAMOUNT trial was a phase II, double-blind study com-
MRAs for potential reduction in hospitalization. paring sacubitril/valsartan with valsartan in 308 patients for
36 weeks (Solomon 2017). Sacubitril/valsartan provided fur-
β-Blockers ther NT-proBNP reduction compared with valsartan at 12
Little evidence suggests benefit with β-blocker therapy in weeks. These positive findings have led to further investigation
patients with HFpEF, except for nebivolol. The SENIORS study within the phase III PARAGON-HF trial, which has completed
evaluated nebivolol in patients 70 and older regardless of EF enrollment (estimated completion May 2019) (Solomon 2017).
with a composite of all-cause mortality or hospital admis-
sion (Flather 2005). After 21 months of therapy, the primary Sildenafil
outcome occurred in 332 patients (31.1%) receiving nebivo- Initial findings with sildenafil use in HFpEF were significant
lol and in 375 patients (35.3%) receiving placebo (p=0.039). improvements in mean pulmonary arterial pressure, right
The primary outcome was not influenced by EF, suggesting atrial pressure, and right ventricular function, with an addi-
that nebivolol can play a role in patients with HFpEF. Even tional improvement in quality of life in 44 patients (Guazzi
though the SENIORS study was not powered to show nebiv- 2011). To validate these findings, a multicenter randomized,
olol’s effect in EF subgroups, the authors maintain that given double-blind, placebo-controlled study was conducted. The
similar hazard ratios with no apparent treatment interaction, RELAX study (n=216 patients) investigated sildenafil 20 mg
the benefit is similar in patients with impaired and preserved three times daily (target dose 60 mg three times daily) com-
EF (van Veldhuisen 2009). Although the European Society of pared with placebo for improvement in exercise capacity or
Cardiology provides commentary regarding nebivolol’s role in clinical status (Redfield 2013). At the end of the study, no
reduced deaths and CV hospitalizations, the ACC/AHA guide- demonstrable difference had occurred in the investigated
lines only recommend this agent for patients with hyper- outcomes. In line with these results, a subsequent study
tension as a treatment option for blood pressure lowering specifically evaluating HFpEF and concomitant pulmonary
(Ponikowski 2016). hypertension also reported that sildenafil within this patient
subset with HF provided no benefits with respect to clinical
Diuretics or exercise parameters (Hoendermis 2015). Given these data,
Diuretic therapy should be used to help improve symptoms sildenafil should not be used solely to improve HFpEF.
associated with congestion and volume overload. Man-
agement should be influenced by fluctuations in daily body Nitrates
weights, symptoms, and electrolyte status. Although loop Nitrates are generally not recommended in patients with
diuretic therapy is typically used in these patients, thia- HFpEF, given the lack of evidence. In addition, within the
zide-like diuretics (e.g., metolazone) can be used to augment NEAT-HFpEF study, patients randomly assigned to isosor-
and help maximize fluid removal in those with difficult-to-man- bide mononitrate (target dose 60–120 mg/day) had a nonsig-
age fluid balances. Caution should be used with aggressive nificant trend toward lower daily activity, with a significant
diuresis because clinically significant hypotension can result decrease in hours of activity per day. Reduction in activity
from the relationship between LV diastolic pressure and vol- was shown irrespective of isosorbide mononitrate dosing,
ume status. Specifically, patients with HFpEF are extremely with significant reductions in activity levels with increasing

PSAP 2019 BOOK 1 • Cardiology 102 Heart Failure


doses. Given these findings, use of nitrates for symptomatic controversial. Meta-analyses showed a 21% increased risk
improvement for HFpEF is not currently recommended. of mortality (n=318,000 patients) and a 29% increase in mor-
tality (n=235,000 patients) when patients were treated with
HF THERAPIES WITH LIMITED digoxin therapy (Vamos 2015). This is in contrast to a subse-
BENEFIT OR POTENTIAL HARM quent meta-analysis that performed a post hoc defined sen-
Because of the burden of HF on the health care system, addi- sitivity analysis and showed no significant effect on mortality
tional medications have been evaluated to circumvent the in patients with HF and AF (Ziff 2015). Given the controversy
harmful aspects of worsening HF. However, many of the medi- surrounding digoxin, the current guidelines recommend that
cations described in the text that follows have failed to provide it be used for heart rate reduction in patients with AF when
additional, if any, benefit toward improving HF symptoms, hos- other therapeutic options cannot be used.
pitalizations, or mortality. These agents should thus be used Digoxin dosing is 0.125–0.25 mg/day, but a starting dose
cautiously, when alternative options have been exhausted, of 0.125 mg/day is recommended, given the propensity for
and only when the potential benefits outweigh the risk. increased toxicity with the higher dose and the therapeu-
tic benefit with lower serum concentrations. Further dose
Digoxin reductions are warranted in patients with impaired renal
Digoxin is one of the oldest medications within the HF phar- function or concomitant drug interaction (e.g., amiodarone)
macotherapy spectrum, but its use remains controversial and in older patients. Use of digoxin concentrations to guide
(Table 4). Use of digoxin in patients with mild to moderate therapy has been debated because the literature supporting
HF is associated with improved symptoms and exercise tol- this practice is limited to retrospective assessments. Most
erance but has failed to show a mortality benefit. In addition, clinicians target drug concentrations of 0.5–0.9 ng/mL,
digoxin’s true efficacy is difficult to determine, given the lack given the correlation between digoxin concentration and sur-
of therapies used within the DIG trial (e.g., β-blockers, MRAs, vival, as noted in a post hoc analysis of the DIG trial (Digitalis
and device therapy [ICD, CRT]). Therefore, digoxin should only Investigation Group 1997). Patients with digoxin concentra-
be considered once GDMT has been optimized. tions greater than 1.2 ng/mL had increased mortality, which
Atrial fibrillation is often concomitant in patients with HFrEF, led to the recommendation to target lower serum digoxin
but given the limited evidence, digoxin use in AF remains concentrations.

Table 4. Comparison of Digoxin and Ivabradine

Digoxin Ivabradine

Indication HFrEF HFrEF with heart rate ≥ 70 beats/min on maximally


SVT or AF tolerated β-blocker

Mechanism of HFrEF: Inhibition of Na/K channel leading to Inhibition of the If channels within the sinoatrial node
action increased intracellular calcium and contractility leading to prolonged diastolic depolarization and a
Arrhythmias: AV nodal suppression reduction in heart rate

Dosing 0.125–0.25 mg QD 2.5–7.5 mg BID

Adverse effects GI, CNS, arrhythmias AF, bradycardia

Consideration Renal function Contraindicated in AF


Drug interactions Patient needs to be stable in NSR
Digoxin concentrations

Clinical benefits HFrEF: Improvement in symptoms and exercise HFrEF: Reduce the risk of hospitalizations in patients
tolerance with reduction in HF hospitalizations with stable HF with heart rate ≥ 70 beats/min in NSR

Appropriate use Consider in symptomatic patients who are Patient with stable HFrEF in NSR with heart rate ≥
currently treated with ACEI/ARB and β-blocker 70 beats/min on maximally tolerated β-blocker
May be considered in patients with symptomatic
HFrEF with concomitant AF when alternative
options cannot be pursued

AF = atrial fibrillation; AV = atrioventricular; HFrEF = heart failure with reduced ejection fraction; NSR = normal sinus rhythm; SVT =
supraventricular tachycardia.

PSAP 2019 BOOK 1 • Cardiology 103 Heart Failure


n-3 Polyunsaturated Fatty Acids Renin Inhibitors
Mainly used for their benefits on hypertriglyceridemia, n-3 Renin serves as the starting point within the RAAS pathway
polyunsaturated fatty acids (PUFAs) had a small benefit in by cleaving the peptide angiotensinogen into AngI, which is
patients with HF (Tavazzi 2008). In a trial evaluating around converted to AngII. Inhibition of renin would theoretically be
7000 patients with HF in cardiology and internal medicine an additional target to decrease circulating AngII concen-
centers in Italy, patients were randomized to n-3 PUFAs 1 g/ trations; however, the direct renin inhibitor aliskiren failed to
day or placebo with a primary composite end point of time to reduce CV deaths or rehospitalizations for patients at 6 or 12
death, or time to death or hospital admission related to a CV months. Adding aliskiren to the current RAAS agents used in
cause. Within the intention-to-treat analysis, death from any the ASTRONAUT trial led to increased episodes of hyperkale-
cause was slightly lower in the n-3 PUFA group (27%) than in mia, hypotension, and renal impairment failure (Gheorghiade
placebo (29%, p=0.041). Despite these findings, the authors 2013). Given the increased risk without benefits, aliskiren is
also noted that the number needed to treat to avoid one death not currently recommended for patients with HF.
was 56 patients requiring treatment for a median of 3.9 years.
Given the currently available information, the use of PUFA are Statins
deemed safe in HF patients and may provide some benefit. Statin therapy is widely used in various clinical scenarios
for its beneficial effects on inflammation, oxidative stress,
Oral Anticoagulants and Antiplatelets and immunologic activities. After reported benefits from
In patients with HF having a clear indication for anticoagu- small observational or post hoc analyses within patients
lation therapy (AF, history of a thromboembolic event), anti- with HF, two randomized controlled trials studied the poten-
coagulation therapy should be individualized depending on tial usefulness of statin therapy. Rosuvastatin was evalu-
the clinical scenario. In patients without a clear indication ated for its potential long-term benefits in patients receiving
for anticoagulation therapy, the decision to initiate therapy is optimal GDMT. The CORONA and GISSI-HF trials failed to
more controversial. Patients with HFrEF have an inherently show any merit for the routine use of statin therapy in HFrEF
increased risk of blood stasis because of hypokinetic or aki- (Kjekshus 2007; Tavazzi 2008). In the absence of a compel-
netic heart chambers, which can lead to thrombus develop- ling indication, statin use alone in HF has not been proven
ment. In clinical studies, patients with HFrEF remained at beneficial.
low risk (1%–3%) of thromboembolism, even those with a
severely reduced EF. Initiation of warfarin in these cases has Calcium Channel Blockers

had mixed results, with a reduction in major CV events and Patients should avoid non-dihydropyridine CCBs (diltiazem,
death in some trials that was not replicated in other cases. verapamil) because of their potential harm in those with symp-
To further ascertain potential benefits of anticoagulation in tomatic HFrEF. Patients in the Multicenter Diltiazem Post
these high risk patients, the COMMANDER-HF study evalu- Infarction Trial (MDPIT) initiated on diltiazem were at a sig-
ated rivaroxaban in HFrEF patients with no history of AF. Par- nificant risk of developing congestive HF compared with pla-
ticipants were randomized to rivaroxaban (2.5 mg twice daily) cebo (p=0.0017) (Goldstein 1991). The dihydropyridine CCBs
or placebo with the hypothesis that the study medication amlodipine and felodipine appear to be safe in both patient
would lead to a reduction in death and CV events. Following a subsets with HF, but given their lack of benefit, they should be
median follow-up period of 21.1 months, the authors found no reserved for select patients already receiving GDMT (HFrEF)
difference in the rate of death, MI or stroke (rivaroxaban: 25% or in patients whose hypertension is inadequately controlled
vs placebo: 26.2%, HR 0.94, 95% CI 0.84-1.05; p=0.27). There with an ACEI/ARB or β-blocker.
was no difference in the primary composite safety outcome
OTC Medications
(fatal bleeding or bleeding into a critical space) but a signifi-
cant increase in the risk of ISTH-defined major bleeding was Nonsteroidal Anti-inflammatory Drugs
noted within the rivaroxaban cohort (3.3% vs 2%, HR 1.68, 95% Nonsteroidal anti-inflammatory drugs are commonly used
CI 1.18-2.39; p=0.003). Overall, current data do not support the in the general population because of their anti-inflammatory
routine use of anticoagulation in patients with HFrEF and no and analgesic properties. Inhibition of renal prostaglandins
other compelling indication. Of greater concern is the risk of leads to vasoconstriction of the afferent arteriole and sub-
complications, specifically the risk of major bleeding without sequent increase in sodium and water resorption. Although
a significant benefit for therapy. generally well tolerated in healthy individuals, NSAIDs should
As with warfarin, no available evidence suggests any ben- be avoided in patients with HF, given their propensity to
efit of antiplatelet therapy in patients with HF without con- worsen kidney function and raise blood pressure (through
comitant disease necessitating therapy (e.g., coronary sodium and water retention), thereby opposing the diuretic
artery disease). Because of the associated increased risk of therapy. Heart failure exacerbations and increased mortality
GI bleeding when using antiplatelet therapy, its routine use, have occurred in observational cohort studies that evaluated
except in ischemic cardiomyopathy, is not recommended. selective or nonselective NSAID therapy.

PSAP 2019 BOOK 1 • Cardiology 104 Heart Failure


Decongestants ACEI/ARB/ARNI, β-blocker, and MRA therapy were not pre-
Pseudoephedrine is an OTC medication that is often used as scribed for 27%, 33%, and 67% respectively. Of these patients,
a decongestant in combination with select cough and cold or target doses were seldom achieved with an ACEI/ARB (17%),
anti-allergy medications. In addition to pseudoephedrine’s pri- an ARNI (14%), and a β-blocker (28%), whereas patients
mary role of vasoconstriction within the respiratory mucosa receiving MRAs more commonly received the doses achieved
and relaxation of the bronchial tree, its adrenergic activation in clinical studies (77%). Using adjusted models, lower medi-
can lead to systemic responses, increasing contractility and cation dosing or use occurred in patients of older age, those
pulse. Patients with HF may not tolerate the added catechol- with lower blood pressure, hospitalized patients, and patients
amine surge, which can worsen HF symptoms. Moreover, stim- with renal insufficiency.
ulation of α-receptors in the vasculature can increase afterload, As medication experts, pharmacists are pivotal in this
which may be difficult for a failing heart to overcome, leading to challenging puzzle of HF medication optimization. Pharma-
reduced cardiac output. Given the concerns for worsening HF, cists must work with clinicians to ensure that appropriate HF
pseudoephedrine is generally not recommended for patients medications are initiated with strategies to titrate toward tar-
with HF, especially those with symptomatic HF. get doses that have shown beneficial morbidity and mortal-
ity results.
CONCLUSION
Preventing hospital readmissions through optimizing transi- REFERENCES
tions of care is crucial to improving HF outcomes in addition Allen LA, Stevenson LW, Grady KL, et al. Decision making
to reducing hospital readmissions, morbidity, mortality, and in advanced heart failure: a scientific statement from the
costs. Use of pharmacy services as part of a multidisciplinary American Heart Association. Circulation 2012;125:1928-52.
HF service has been shown beneficial for medication recon- Andersson C, Olesen JB, Hansen PR, et al. Metformin treat-
ciliation and education and has reduced medication errors as ment is associated with a low risk of mortality in diabetic
well as hospital costs. However, even with the proven bene- patients with heart failure: a retrospective nationwide
fits of HF teams, recent findings within the CHAMP-HF study cohort study. Diabetologia 2010;53:2546-53.
suggest that more involvement is needed (Greene 2018). To Bardy GH, Lee KL, Mark DB, et al. Amiodarone or an implant-
identify potential gaps between guideline recommendations able cardioverter-defibrillator for congestive heart failure.
and real-world management, the CHAMP-HF study character- N Engl J Med 2005;352:225-37.
ized medication use and doses within clinical practice. Spe-
Benjamin EJ, Virani SS, Callaway CW, et al. Heart disease
cifically evaluating patients with HF in the outpatient setting, and stroke statistics-2018 update: a report from the Ameri-
can Heart Association. Circulation 2018;137:e67-e492.

Cleland JG, Daubert JC, Erdmann E, et al. The effect of car-


Practice Points
diac resynchronization on morbidity and mortality in heart
• In patients with risk factors for HF, appropriate measures failure. N Engl J Med 2005;352:1539-49.
should be taken to aggressively manage comorbidities to
prevent the onset of HF. ClinicalTrials.gov [homepage on the Internet]. A Service of
• Patients with HF should be assessed and actively treated the U.S. National Institutes of Health. Study to Compare
for concomitant diabetes, hypertension, or anemia to help Ferric Carboxymaltose with Placebo in Patients with Acute
improve quality of life (anemia) and reduce CV complica- Heart Failure and Iron Deficiency (AFFIRM-AHF).
tions (diabetes).
ClinicalTrials.gov [homepage on the Internet]. A Service of
• Ensure that patients with HFrEF are receiving a RAAS
the U.S. National Institutes of Health. Intravenous Iron
inhibitor (ACEI, ARB, or ARNI) in combination with β-block-
Treatment in Patients with Heart Failure and Iron Defi-
ers and MRAs.
ciency: IRONMAN.
• For patients with HFrEF who are currently tolerating an
ACEI/ARB, transition to an ARNI is recommended to reduce de Denus S, O’Meara E,Desai AS, et al. Spironolactone
the risk of mortality. metabolites in TOPCAT – New insights into regional varia-
• Medication therapy should be dose adjusted to maximally tion. N Engl J Med 2017;367:1690-92.
tolerated doses before adding secondary agents (e.g.,
hydralazine/isosorbide, digoxin). Desai AS, Solomon S, Claggett B, et al. Factors associ-
• In patients receiving maximally tolerated β-blockers with ated with noncompletion during the run-in period before
a heart rate of 70 beats/minute or greater (in normal sinus randomization and influence on the estimated bene-
rhythm), ivabradine should be considered. fit of LCZ696 in the PARADIGM-HF Trial. Circ Heart Fail
• In patients who develop HFpEF, spironolactone should be 2016;9:e002735.
considered to reduce the risk of HF hospitalizations.
• Careful consideration should be given when adding medi- Digitalis Investigation Group. The effect of digoxin on mor-
cations with no proven benefits to HF regimens. tality and morbidity in patients with heart failure. N Engl J
Med 1997;336:525-33.

PSAP 2019 BOOK 1 • Cardiology 105 Heart Failure


Doust JA, Pietrzak E, Dobson A, et al. How well does B-type Ledwidge M, Gallagher J, Conlon C, et al. Natriuretic pep-
natriuretic peptide predict death and cardiac events tide-based screening and collaborative care for heart fail-
in patients with heart failure: systematic review. BMJ ure: the STOP-HF randomized trial. JAMA 2013;310:66-74.
2005;330:625.
Lip GY, Skjoth F, Overvad K, et al. Blood pressure and prog-
Flather MD, Shibata MC, Coats AJ, et al. Randomized trial to nosis in patients with incident heart failure: the diet,
determine the effect of nebivolol on mortality and cardio- cancer and health (DCH) cohort study. Clin Res Cardiol
vascular hospital admission in elderly patients with heart 2015;104:1088-96.
failure (SENIORS). Eur Heart J 2005;26:215-25.
Logeart D, Thabut G, Jourdain P, et al. Predischarge B-type
Ghali JK, Anand IS, Abraham WT, et al. Randomized dou- natriuretic peptide assay for identifying patients at high
ble-blind trial of darbepoetin in patients with symptomatic risk of re-admission after decompensated heart failure.
heart failure and anemia. Circulation 2008;117:526-35. JACC 2004;43:635-41.

Gheorghiade M, Bohm M, Greene SJ, et al. Effect of aliskiren Maisel AS, Krishnaswamy P, Nowak RM, et al. Rapid mea-
on postdischarge mortality and heart failure readmissions surement of B-type natriuretic peptide in the emergency
among patients hospitalized for heart failure: the ASTRO- diagnosis of heart failure. N Engl J Med 2002;347:161-7.
NAUT randomized trial. JAMA 2013;309:1125-35.
Marso SP, Bain SC, Consoli A, et al.; for the SUSTAIN-6
Goldstein RE, Boccuzzi SJ, Cruess D, et al. Diltiazem Investigators. Semaglutide and cardiovascular out-
increases late-onset congestive heart failure in post comes in patients with type 2 diabetes. N Engl J Med
infarction patients with early reduction in ejection frac- 2016;375:1834-44.
tion. The Adverse Experience Committee; and the Mul-
ticenter Diltiazem Post-infarction Research Group. Marso SP, Daniels GH, Brown-Frandsen K, et al.; for the
Circulation 1991;83:52-60. LEADER Steering Committee on behalf of the LEADER
Trial Investigators. Liraglutide and cardiovascular out-
Green JB, Bethel MA, Armstrong PW, et al.; for the TECOS comes in type 2 diabetes. N Engl J Med 2016b;375:311-22.
Study Group. Effect of sitagliptin on cardiovascular out-
comes in type 2 diabetes. N Engl J Med 2015;373:232-42. Martin RI, Pogoryelova O, Koref MS, et al. Atrial fibrillation
associated with ivabradine treatment: meta-analysis of
Greene SJ, Butler J, Albert NM, et al. Medical therapy randomised controlled trials. Heart 2014;100:1506-10.
for heart failure with reduced ejection fraction: the
CHAMP-HF Registry. J Am Coll Cardiol 2018;72:351-66. Massie BM, Carson PE, McMurray JJ, et al. Irbesartan in
patients with heart failure and preserved ejection fraction.
SPRINT ResearchGroup, Wright JT Jr, Williamson JD, et al. A N Engl J Med 2008;359:2456-67.
randomized trial of intensive versus standard blood pres-
sure control. N Engl J Med 2015;373:2103-16. McMurray JJ, Packer M, Desai AS, et al. Angiotensin-nepri-
lysin inhibition versus enalapril in heart failure. N Engl J
Guazzi M, Vicenzi M, Arena R, et al. Pulmonary hypertension Med 2014;371:993-1004.
in heart failure with preserved ejection fraction: a target of
phosphodiesterase-5 inhibition in a 1-year study. Circula- Moss AJ, Hall WJ, Cannom DS, et al. Cardiac-resynchroni-
tion 2011;124:164-74. zation therapy for the prevention of heart-failure events.
N Engl J Med 2009;361:1329-38.
Heidenreich PA, Albert NM, Allen LA, et al. Forecasting the
impact of heart failure in the United States: a policy state- Neal B, Perkovic V, Mahaffey KW, et al. Canagliflozin and car-
ment from the American Heart Association. Circ Heart Fail diovascular and renal events in type 2 diabetes. N Engl J
2013;6:606-19. Med 2017;377:644-57.

Hoendermis ES, Liu LC, Hummel YM, et al. Effects of silde- Nizamic T, Murad H, Allen LA, et al. Ambulatory inotrope infu-
nafil on invasive haemodynamics and exercise capacity in sions in advanced heart failure: a systematic review and
heart failure patients with preserved ejection fraction and meta-analysis. JACC Heart Fail 2018;6:757-67.
pulmonary hypertension: a randomized controlled trial. O’Connor CM, Starling RC, Hernandez AF, et al. Effect of
Eur Heart J 2015;36:2565-73. nesiritide in patients with acute decompensated heart fail-
ONTARGET Investigators, Yusuf S, Teo KK, et al. Telmisar- ure. N Engl J Med 2011;365:32-43.
tan, ramipril, or both in patients at high risk for vascular Okonko DO, Mandal AK, Missouris CG, et al. Disordered iron
events. N Engl J Med 2008;358:1547-59. homeostasis in chronic heart failure: prevalence, predic-
Jankowska EA, Tkaczyszyn M, Suchocki T, et al. Effects of tors, and relation to anemia, exercise capacity, and sur-
intravenous iron therapy in iron-deficient patients with vival. J Am Coll Cardiol 2011;58:1241-51.
systolic heart failure: a meta-analysis of randomized con- Packer M, McMurray JJ, Desai AS, et al. Angiotensin recep-
trolled trials. Eur J Heart Fail 2016;18:786-95. tor neprilysin inhibition compared with enalapril on the
Kjekshus J, Apetrei E, Barrios V, et al; for the CORONA Group. risk of clinical progression in surviving patients with heart
Rosuvastatin in older patients with systolic heart failure. failure. Circulation 2015;131:54-61.
N Engl J Med 2007;357:2248-61.

PSAP 2019 BOOK 1 • Cardiology 106 Heart Failure


Pfeffer MA, Claggett B, Assmann SF, et al. Regional variation in the African-American heart failure trial. Circulation
in patients and outcomes in the treatment of preserved 2007;115:1747-53.
cardiac function heart failure with and aldosterone antag-
onist (TOPCAT) trial. Circulation 2015;131:34-42. Taylor CJ, Ryan R, Nichols L, et al. Survival following a
diagnosis of heart failure in primary care. Fam Pract
Pitt B, Pfeffer MA, Assmann SF, et al. Spironolactone for 2017;34:161-8.
heart failure with preserved ejection fraction. N Engl J
Med 2014;370:1383-92. Vamos M, Erath JW, Hohnloser SH. Digoxin-associated mor-
tality: a systematic review and meta-analysis of the litera-
Ponikowski P, Voors AA, Anker SD, et al. 2016 ESC guidelines ture. Eur Heart J 2015;36:1831-8.
for the diagnosis and treatment of acute and chronic heart
failure: the task force for the diagnosis and treatment of van Veldhuisen DJ, Cohen-Solal A, Bohm M, et al. Beta-block-
acute and chronic heart failure of the European Society of ade with nebivolol in elderly heart failure patients with
Cardiology (ESC). Developed with the special contribution impaired and preserved left ventricular ejection fraction:
of the Heart Failure Association (HFA) of the ESC. Eur J data from SENIORS (study of effects of nebivolol inter-
Heart Fail 2016;18:891-975. vention on outcomes and rehospitalization in seniors with
heart failure). JACC 2009;53:2150-8.
Rahimi K, Bennett D, Conrad N, et al. Risk prediction in
patients with heart failure: a systematic review and analy- Vardeny O, Claggett B, Kachadourian J, et al. Incidence, pre-
sis. JACC Heart Fail 2014;2:440-6. dictors, and outcomes associated with hypotensive epi-
sodes among heart failure patients receiving sacubitril/
Redfield MM, Chen HH, Borlaug BA, et al. Effect of phospho- valsartan or enalapril: the PARADIGM-HF Trial (prospec-
diesterase-5 inhibition on exercise capacity and clinical tive comparison of angiotensin receptor neprilysin inhib-
status in heart failure with preserved ejection fraction: a itor with angiotensin-converting enzyme inhibitor to
randomized clinical trial. JAMA 2013;309:1268-77. determine impact on global mortality and morbidity in
heart failure). Circ Heart Fail 2018;11:e004745.
Scirica BM, Bhatt DL, Braunwald E, et al.; for the SAVOR-TIMI
53 Steering Committee and Investigators. Saxagliptin and Velazquez EJ, Morrow DA, DeVore AD, et al; for the PIO-
cardiovascular outcomes in patients with type 2 diabetes NEER-HF Investigators. Angiotensin-neprilysin inhibi-
mellitus. N Engl J Med 2013;369:1317-26. tion in acute decompensated heart failure. N Engl J Med
2018;Nov 11 [Epub ahead of print].
Sears SF, Hauf JD, Kirian K, et al. Posttraumatic stress and
the implantable cardioverter-defibrillator patient: what the White WB, Cannon CP, Heller SR, et al.; for the EXAM-
electrophysiologist needs to know. Circ Arrhythm Electro- INE Investigators. Alogliptin after acute coronary syn-
physiol 2011;4:242-50. drome in patients with type 2 diabetes. N Engl J Med
2013;369:1327-35.
Solomon SD, Rizkala AR, Gong J, et al. Angiotensin receptor
neprilysin inhibition in heart failure with preserved ejec- Wiviott SD, Raz I, Bonaca MP, et al.; for the DECLARE-TIMI 58
tion fraction: rationale and design of the PARAGON-HF Investigators. Dapagliflozin and cardiovascular outcomes
Trial. JACC Heart Fail 2017;5:471-82. in type 2 diabetes. N Engl J Med 2018; Nov 10 [Epub ahead
of print].
Stades AM, Heikens JT, Erkelens DW, et al. Metformin and
lactic acidosis: cause or coincidence? A review of case Yancy CW, Jessup M, Bozkurt B, et al. 2017 ACC/AHA/HFSA
reports. J Intern Med 2004;255:179-87. focused update of the 2013 ACCF/AHA guideline for the
management of heart failure: a report of the American Col-
Stavrakis S, Lazzara R, Thadani U. The benefit of cardiac lege of Cardiology/American Heart Association Task Force
resynchronization therapy and QRS duration: a meta-anal- on Clinical Practice Guidelines and the Heart Failure Soci-
ysis. J Cardiovasc Electrophysiol 2012;23:163-8. ety of America. Circulation 2017;136:e137-61.
Swedberg K, Komajda M, Bohm M, et al. Ivabradine and out- Yusuf S, Pfeffer MA, Swedberg K, et al. Effects of candesar-
comes in chronic heart failure (SHIFT): a randomised pla- tan in patients with chronic heart failure and preserved
cebo-controlled study. Lancet 2010;376:875-85. left-ventricular ejection fraction: the CHARM-Preserved
Tanboga IH, Topcu S, Aksakal E, et al. The risk of atrial fibril- trial. Lancet 2003;362:777-81.
lation with ivabradine treatment: a meta-analysis with trial Zannad F, McMurray JJ, Krum H, et al. Eplerenone in patients
sequential analysis of more than 40000 patients. Clin Car- with systolic heart failure and mild symptoms. N Engl J
diol 2016;39:615-20. Med 2011;364:11-21.
Tavazzi L, Maggioni AP, Marchioli R, et al. Effect of n-3 poly- Ziff OJ, Lane DA, Samra M, et al. Safety and efficacy of
unsaturated fatty acids in patients with chronic heart digoxin: systematic review and meta-analysis of observa-
failure (the GISSI-HF trial): a randomised, double-blind, tional and controlled trial data. BMJ 2015;351:h4451.
placebo-controlled trial. Lancet 2008;37:1223-30.
Zinman B, Wanner C, Lachin JM, et al.; for the EMPA-REG
Taylor AL, Ziesche S, Yancy CW, et al. Early and sustained OUTCOME Investigators. Empagliflozin, cardiovascular
benefit on event-free survival and heart failure hospi- outcomes, and mortality in type 2 diabetes. N Engl J Med
talization from fixed-dose combination of isosorbide 2015;373:2117-28.
dinitrate/hydralazine: consistency across subgroups

PSAP 2019 BOOK 1 • Cardiology 107 Heart Failure


Self-Assessment Questions
16. A 43-year-old man (weight 240 lb) with no significant 17. Which one of the following best describes G.K.’s NYHA
medical history routinely sees his primary physician functional class?
for an annual examination because of an ongoing con-
A. I
cern for elevated blood pressure. He is reluctant to start
B. II
taking blood pressure medications, and given his blood
C. III
pressure of 135/86 mm Hg and lack of additional risk fac-
D. IV
tors, his physician agrees to first try diet, exercise, and
lifestyle changes. Subsequently, the patient presents to 18. G.K. is diuresed to his “normal” weight, and after his
the ED with fever and difficulty breathing. Pneumonia HF medications are titrated, he is almost ready for dis-
is initially diagnosed, and the patient is discharged on charge. His current weight is 79 kg, with correlating vital
antibiotics and steroids. However, the patient does not signs as follows: heart rate 65 beats/minute, blood pres-
improve and is sent to the hospital for a further workup. sure 100/68 mm Hg, and respiratory rate 15 breaths/min-
Chest radiography reveals pulmonary edema versus ute with 100% saturation. Relevant laboratory values are
infection. Given his ongoing shortness of breath, car- K 4.6 mEq/L, SCr 1.1 mg/dL (baseline 0.9 mg/dL), Na 141
diology is consulted. An ECHO is unremarkable, with a mEq/L, and BNP 600 pg/mL. His current drug regimen is
reported left ventricular ejection fraction (LVEF) greater aspirin 81 mg daily, carvedilol 12.5 mg twice daily, ator-
than 55%. The patient has no evidence of dyspnea on vastatin 80 mg daily, lisinopril 20 mg daily, and bumeta-
exertion, orthopnea, or lower-extremity swelling. Which nide 2 mg daily. Which one of the following is best to add
one of the following best classifies this patient with to G.K.’s regimen before discharge?
respect to ACC/AHA and NYHA? A. Hydralazine/isosorbide (37.5 mg/20 mg combination
A. ACC/AHA stage C and NYHA class II tablet) three times daily
B. ACC/AHA stage A and NYHA class III B. Digoxin 0.25 mcg daily
C. ACC/AHA stage B and NYHA class IV C. Spironolactone 12.5 mg daily
D. ACC/AHA stage A with no accompanying NYHA D. Sacubitril/valsartan 24/26 mg twice daily
classification 19. A 54-year-old white man has a medical history signifi-
cant for nonischemic cardiomyopathy (NICM) caused
Questions 17 and 18 pertain to the following case. by viral myocarditis. He has no other significant medi-
G.K. is a 64-year-old African American man with a medical cal history and currently has been treated on an HF regi-
history of ischemic cardiomyopathy (EF 25%) because of men consistent with sacubitril/valsartan 2426 mg twice
an MI in 1999 requiring three-vessel coronary artery bypass daily, metoprolol succinate 50 mg daily, spironolactone
grafting. He was initiated, and has remained, on appropriate 25 mg daily, and as-needed bumetanide 1 mg daily. He is
medical therapy that includes aspirin 81 mg daily, carvedilol seen in the clinic and admitted to the hospital with fail-
12.5 mg twice daily, atorvastatin 80 mg daily, and lisinopril ure to thrive and reduced ability to perform daily activi-
10 mg daily. G.K. continues to be seen by cardiology but indi- ties. His weight has decreased from 74 kg (3 months ago)
cates that he has recently been more short of breath, requir- to 64 kg as his admission weight. His admission labora-
ing 2 or 3 pillows at night to help him sleep. He denies the tory values are unremarkable and unchanged from pre-
use of diuretic therapy recently because there is no overt vious clinic visits: Hgb 8.9 g/dL, SCr 1.3 mg/dL, and K
lower-extremity edema. G.K. checks his weight three or four 4.1 mEq/L, with NT-proBNP 335 pg/mL and serum iron
times per week; until a week ago, his weight had remained 19 mcg/dL (range 50–170 mcg/dL). His vital signs on
around 84 kg, but since then, it has increased to 90 kg. He admission are heart rate 75 beats/minute, blood pres-
denies any change in diet and continues trying to exercise; sure 115/76 mm Hg, and respiratory rate 17 breaths/min-
however, it is becoming more difficult. G.K. cannot walk up a ute with 98% saturation. Given his decreased activity
flight of steps without stopping and can only walk about the level and weight loss, which one of the following is best
one-half the length of the school track. Today’s vital signs to recommend for this patient?
include heart rate 83 beats/minute, blood pressure 104/67 A. Initiate intravenous iron sucrose 200 mg daily x 5
mm Hg, and respiratory rate 19 breaths/minute with Sao2 days.
95%. G.K.’s basic metabolic panel is unremarkable, but his B. Give darbepoetin.
NT-proBNP is significantly elevated at 6000 pg/mL (baseline C. Reduce metoprolol succinate to 25 mg daily.
540 pg/mL). D. Initiate continuous intravenous milrinone 0.25 mcg/
kg/minute.

PSAP 2019 BOOK 1 • Cardiology 108 Heart Failure


20. A 35-year-old African American man recently received 21. The resident treating F.G. comes to you for advice regard-
a diagnosis of HF because of ischemia (history of an ing the patient’s pharmacotherapy. Which one of the fol-
ST-segment elevation MI). The patient underwent per- lowing is best to recommend first for F.G.’s treatment?
cutaneous coronary intervention with drug-eluting stent
A. Lisinopril
placement to his left anterior descending and left circum-
B. Carvedilol
flex arteries (x 2) and was initiated on appropriate med-
C. Bumetanide
ications, including HF medications, because an ECHO
D. Hydrochlorothiazide
revealed an EF less than 35%. He is seen in the clinic
with worsening HF symptoms and describes increased 22. After being initiated on appropriate HF therapy, F.G. is
weight gain, bloating, and general fatigue. Physical discharged on the following medication regimen: lisino-
examination reveals cool, clammy extremities, with jugu- pril 10 mg daily, carvedilol 6.25 mg twice daily, spirono-
lar venous distention and an elevated heart rate. He has lactone 12.5 mg daily, and torsemide 20 mg daily with an
been adherent to his medications, which include losar- extra tablet in the afternoon as needed for weight gain.
tan 50 mg daily, metoprolol succinate 50 mg daily, spi- Six months later, F.G. presents to the clinic with the fol-
ronolactone 25 mg daily, aspirin 81 mg daily, ticagrelor lowing medication list: lisinopril 20 mg daily, metoprolol
90 mg twice daily, atorvastatin 80 mg daily, bumetanide 1 succinate 150 mg daily, spironolactone 25 mg daily, and
mg twice daily, and nitroglycerin as needed. He is admit- torsemide 10 mg as needed. Her vital signs on admission
ted to the hospital for concern for HF exacerbation. Vital are blood pressure 114/64 mm Hg, heart rate 68 beats/
signs on admission are heart rate 88 beats/minute, blood minute in normal sinus rhythm, Sao2 99%, and respiratory
pressure 121/67 mm Hg, and respiratory rate 19 breaths/ rate 19 breaths/minute. All laboratory values are within
minute with 100% saturation. To optimize his care, the normal limits except for SCr 1.8 mg/dL. F.G. has worsen-
physician seeks your recommendation for medical ther- ing shortness of breath, with difficulty lying flat at night
apy. Which one of the following is best to recommend for and increased pillows to help her sleep. The resident
this patient? wants to add ivabradine to her drug regimen. Which one
of the following is best to recommend regarding ivabra-
A. Initiate lisinopril 5 mg daily.
dine use in F.G.?
B. Increase losartan 100 mg daily.
C. Initiate isosorbide dinitrate 20 mg and hydralazine A. Initiate 5 mg twice daily.
37.5 mg three times daily. B. Initiate 2.5 mg twice daily.
D. Change losartan to sacubitril/valsartan 24/26 mg C. Initiate 7.5 mg twice daily.
twice daily. D. Therapy is not indicated.
23. A 67-year-old African American woman has longstand-
Questions 21 and 22 pertain to the following case. ing heart failure with reduced ejection fraction (HFrEF)
F.G. is a 55-year-old white woman with a medical history (stage C). Her other medical history consists of diabetes
significant for diabetes and hypertension. Despite her mellitus, hypertension, several genitourinary infections,
medical history, she does not actively take medications and a remote history of intravenous drug use (more than
and has not been adherent to medical advice or with clinic 20 years ago). The patient’s condition has been well con-
follow-ups. F.G. presents to the ED with profound volume trolled on lisinopril 20 mg daily, carvedilol 25 mg twice
overload, as evidenced by a 10-kg weight gain over the past daily, eplerenone 50 mg daily, and furosemide 20 mg twice
7–10 days; difficulty breathing; 3+ pitting edema in her daily. In the clinic today, she reports feeling well but indi-
lower extremities; and swelling in her abdomen. Her lab- cates that her diet has not been the best and that her blood
oratory results are normal except for Na 131 mEq/L, K 4.8 glucose readings have been a little high. An A1C reading is
mEq/L, SCr 2.4 mg/dL (baseline 1.1 mg/dL), and NT-proBNP 8.1%, significantly increased from 6.5% 6 months ago. She
13,000 pg/mL. Given the concern for cardiac involvement, is currently not receiving any medications for diabetes.
an ECHO is done that reveals severely reduced cardiac Understanding the risks of uncontrolled diabetes (given
function, with an LVEF of less than 20% with an enlarged her family history), she seeks medication therapy to bet-
LV cavity. F.G.’s vital signs are blood pressure 144/84 mm ter prevent cardiovascular (CV) complications. Which one
Hg, heart rate 102 beats/minute, Sao2 98%; and respiratory of the following is best to recommend for this patient?
rate 20 breaths/minute. She is admitted to the HF service A. Metformin 500 mg twice daily
for treatment. B. Empagliflozin 10 mg daily
C. Saxagliptin 2.5 mg daily
D. Glyburide 10 mg daily

PSAP 2019 BOOK 1 • Cardiology 109 Heart Failure


24. T.A., a 74-year-old man, presents to the ED with the chief overload. H.M. was treated with intravenous diuretics and
concern of a racing heart. He describes it as if his heart later discharged. His most recent EF was 15% during hospital-
is beating through his chest. Initial heart rate shows that ization. His vital signs today are heart rate 86 beats/minute,
he is tachycardiac at 114–132 beats/minute, with blood blood pressure 114/58 mm Hg, respiratory rate 17 breaths/
pressure 104/65 mm Hg. T.A. states that he has long- minute, and Sao2 99%. H.M.’s home drugs include losartan 50
standing HFrEF with a current drug regimen of sacubi- mg daily, metoprolol succinate 100 mg daily, spironolactone
tril/valsartan 49/51 mg twice daily, bisoprolol 2.5 mg 25 mg daily, potassium chloride 20 mEq twice daily, atorvas-
daily (intolerant of higher doses), spironolactone 25 tatin 80 mg daily, aspirin 81 mg daily, and bumetanide 2 mg
mg twice daily, and ivabradine 5 mg twice daily. He was daily. The resident treating H.M. decides to initiate ivabradine
recently given a diagnosis of diabetes and instructed to 5 mg twice daily.
start empagliflozin, given the benefits of empagliflozin
26. H.M. is seen 2 weeks after ivabradine initiation and
for patients with his condition. His laboratory values
reports significant fatigue and lightheadedness. His vital
are all within normal limits, with SCr 0.89 mg/dL and K
signs are heart rate 61 beats/minute, blood pressure
4.6 mEq/L. Given his heart rate, an ECG is done, which
101/54 mm Hg, and respiratory rate 16 breaths/minute.
reveals the patient is likely in atrial fibrillation (AF). T.A. is
Which one of the following is best to recommend regard-
given a dose of intravenous metoprolol 5 mg to help con-
ing ivabradine for H.M.?
trol his heart rate and is admitted for further evaluation.
T.A.’s care team is searching for treatment options for his A. Continue current regimen.
new AF. Which one of the following is best to recommend B. Decrease to 2.5 mg twice daily.
for T.A.? C. Increase to 7.5 mg twice daily.
D. Discontinue.
A. Digoxin 125 mcg daily, goal 0.5–0.9 ng/mL
B. Digoxin 125 mcg every other day, goal greater than 27. H.M. continues to be seen by the HF service as an out-
1.2 ng/mL patient. Overall, he is doing well but is beginning to have
C. Diltiazem IR 30 mg three times daily more frequent hospitalizations because of HF. On review-
D. Metoprolol tartrate 25 mg twice daily ing the patient’s medication profile, the attending physi-
cian wants make adjustments to better compensate the
25. A 67-year-old African American man has a medical his-
patient and reduce hospitalizations. Which one of the fol-
tory significant for NICM 2/2 viral myocarditis, hyperten-
lowing is best to recommend for H.M.?
sion, and ventricular tachycardia. His HF is well managed
with sacubitril/valsartan 49–51 mg twice daily, metopro- A. Substitute eplerenone 25 mg daily for
lol succinate 150 mg daily, eplerenone 50 mg daily, and spironolactone.
furosemide 40 mg twice daily. The patient underwent B. Discontinue losartan and initiate sacubitril/
an internal cardiac defibrillator implantation with car- valsartan 24/26 mg twice daily without a washout
diac resynchronization therapy (ICD/CRT) upgrade for period.
his history of ventricular tachycardia. He asks whether C. Add isosorbide dinitrate 20 mg and hydralazine
any OTC medications are safe that might improve his HF. 37.5 mg three times daily.
Which one of the following is best to recommend for this D. Discontinue losartan and initiate irbesartan 300 mg
patient? daily.

A. Fish oil 28. A 45-year-old man has newly diagnosed HF and a med-
B. Aspirin ical history of hypertension and diabetes. His initial
C. Ibuprofen ECHO reveals heart failure with preserved ejection frac-
D. Caffeine tion (HFpEF) (LVEF 55%). His current medication ther-
apy includes hydrochlorothiazide 25 mg daily, metformin
1000 mg twice daily, furosemide 20 mg daily (as needed),
Questions 26 and 27 pertain to the following case.
and aspirin 81 mg daily. His diabetes and hyperten-
H.M. is a 65-year-old white man with a medical history of MI
sion have been well controlled with this regimen, with
(15 years ago) that required stent placements and subse-
an A1C of 6.5% and blood pressure of 114–125 systolic
quent three-vessel bypass surgery. Additional medical his-
and 65–75 diastolic. Important laboratory values include
tory includes diabetes and gout. After his MI, H.M.’s heart
K 4.1 mEq/L and SCr 1.1 mg/dL. The patient reports
function slowly deteriorated until he developed systolic dys-
increased swelling with a 3-lb weight gain over the past
function. He has been seen in the HF clinic for several years
2–3 days, for which he has taken three doses of furo-
and has been doing well with ongoing optimization of his
semide. He denies significant shortness of breath but
HF regimen. The patient reports that he was admitted to the
reports he has to stop more often while walking. He can
hospital for 2 days because of decompensation with volume

PSAP 2019 BOOK 1 • Cardiology 110 Heart Failure


still walk 1–2 miles a day. Which one of the following is C. Initiate hydralazine 25 mg three times daily in
best to recommend for this patient? combination with isordil dinitrate 10 mg three times
daily.
A. Irbesartan 150 mg daily
D. Discontinue lisinopril; wait 36 hours and initiate
B. Isosorbide mononitrate 30 mg ER daily
sacubitril/valsartan 24/26 mg twice daily.
C. Spironolactone 25 mg daily
D. Nebivolol 5 mg daily 30. A 67-year-old woman has a medical history significant
for NICM (HFpEF), hypertension, and diabetes mellitus.
29. A 34-year-old man with no medical insurance has a
She has managed her disease states well and been vigil-
medical history that includes viral myocarditis at age
iant about her diet and exercise. However, her blood pres-
31 leading to HFrEF, drug and alcohol use disorder, and
sure has steadily increased, and today, it is 154/88 mm
hypertension. He has been nonadherent to medical ther-
Hg with heart rate 91 beats/minute. She denies chang-
apy and medications over the past 1–2 years. Overall,
ing her diet or exercise regimen. Her current medica-
he feels well without concerns and presents to the car-
tion regimen consists of aspirin 81 mg daily, fish oil 1 g
diology clinic for a routine clinic visit. His current drug
twice daily, and loratidine 10 mg as needed for allergies.
regimen includes lisinopril 40 mg daily, carvedilol 25 mg
Which one of the following is best to recommend for this
twice daily, spironolactone 50 mg daily, and bumetanide
patient?
1 mg daily, but he states that he has been more compliant
since his last cinic visit. His vital signs on presentation A. Sacubitril/valsartan 24/26 mg twice daily
today are as follows: heart rate 69 beats/minute, blood B. Amlodipine 5 mg daily
pressure 154/95 mm Hg, respiratory rate 15 breaths/min- C. Isosorbide mononitrate ER 60 mg daily
ute, and Sao2 99%; his weight is 65 kg (unchanged from D. Irbesartan 150 mg daily
previous visits). Which one of the following would best
reduce this patient’s risk of complications?
A. Increase carvedilol to 50 mg twice daily.
B. Discontinue lisinopril; wait 36 hours and initiate
sacubitril/valsartan 49/51 mg twice daily.

PSAP 2019 BOOK 1 • Cardiology 111 Heart Failure


Learner Chapter Evaluation: Heart Failure.

As you take the posttest for this chapter, also evaluate the Use the 5-point scale to indicate whether this chapter pre-
material’s quality and usefulness, as well as the achievement pared you to accomplish the following learning objectives:
of learning objectives. Rate each item using this 5-point scale:
29. Apply treatment strategies to reduce the progression of
heart failure (HF) through assessing functional class.
• Strongly agree
30. Evaluate and justify traditional and newer treatment

• Agree strategies for patients with stage C HF, specifically as
• Neutral they relate to the 2017 guideline updates.
• Disagree 31. Develop treatment strategies for patients with stage D
• Strongly disagree HF.
32. Evaluate treatment strategies for potential benefit/harm
18. The content of the chapter met my educational needs.
for the patient with HF with preserved ejection fraction.
19. The content of the chapter satisfied my expectations.
20. The author presented the chapter content effectively.
OTHER COMMENTS
21. The content of the chapter was relevant to my practice 33. Please provide any specific comments relating to any
and presented at the appropriate depth and scope. perceptions of bias, promotion, or advertisement of
22. The content of the chapter was objective and balanced. commercial products.
23. The content of the chapter is free of bias, promotion, or 34. Please expand on any of your above responses, and/or
advertisement of commercial products. provide any additional comments regarding this chapter:
24. The content of the chapter was useful to me.
25. The teaching and learning methods used in the chapter
were effective.
26. The active learning methods used in the chapter were
effective.
27. The learning assessment activities used in the chapter
were effective.
28. The chapter was effective overall.

PSAP 2019 BOOK 1 • Cardiology 112 Heart Failure

You might also like